You are on page 1of 94

INTENTIONAL TORTS

What is a Tort?
1. Legally recognized
2. Wrongful injuring of another
3. Generates right of action against wrong doer
Two Primary Goals
1. Compensating victims (Corrective Justice/Civil Recourse)
2. Guiding conduct towards promotion of safety/protection (Deterrence)
Intentional Torts:
- Battery, Assault, False Imprisonment, IIED and Trespass to Property
- Cannot be committed by accident
- Problem with label, suggest each tort it covers employs the same notion of intent
Four Basic Features:
1. Common Law
2. Grounded in relational (typically bilateral) conduct between a (plaintiff-victim) and a
(defendant-tortfeasor)
3. Two primary goals:
1) Compensating victims for harms corrective justice / civil recourse for wrongs
2) Guiding private conduct towards the promotion of safety / and the protection of
human dignity deterrence
4. Begin from the premise that we let the loss lie where it falls (Holmes), unless there is a
good reason (i.e., a legal basis) for shifting it.
Purposes:
Compensatory rationale
o Designed to make the plaintiff whole
Have to assume correspondence between damages and injuries
Tort system may not be efficient, because it involves high transaction costs
Alternative compensatory systems are available, i.e. first-party insurance
Deterrence
o But not always clear that tort law is the most efficient way of deterring risks
behavior if damages arent foreseeable/predictable, actors might not be able to
take into account potential liability
Corrective justice
o Mechanism by which individuals can recover from the actors who caused their
wrongs
o Idea that tort involves a bilateral relationship between an individual and
wrongdoer, and tort allows the individual to sue the wrongdoer to restore the
original state of affairs
Preservation of the peace

Distributing losses within society


o Tort law about allocating responsibility for risks to parties in the best position to
assume liability
Accountability mechanism
o Way for individuals to hold large corporations/institutions accountable
o Redress for social grievances
Fosters limited gov by empowering individuals
A. Battery
1.
Definition: Intentionally causing another to suffer a harmful or offensive contact
- Either harmfulness of offensiveness will suffice
Elements:
1.
A acts
2.
intending to cause a contact with P;
- Contact: some degree of physicality, extended personality
3.
the contact with P that A intends is of a harmful or offensive type; and
- Objective test: R2 19: offends a reasonable sense of personal dignity
o Offensiveness is often a question for the trier of fact to decide
4.
As act causes P to suffer a contact that is harmful or offensive
2. Harmful or Offensive Touching:
Need not be flesh to flesh (i.e. hit by bullet or bitten by a dog)
Anything connected to the body considered part of person
Does not have to be physical (i.e. blowing smoke in eyes/shockwave)
o Contact is harmful if it injures, disfigures, impairs, or causes pain to
P (R.2d 15) (Broken bones & bodily harms)
-Cecarelli v. Maher(1943) (p. 604): Ds beat P over escorting 3
ladies home from a dance
o Contact is offensive if it would be offensive to a reasonable sense of
personal dignity under the circumstances (R.2d 19)
Ps hypersensitive reaction is insufficient, unless D had
knowledge of the hypersensitivity
-Paul v. Holbrook (p. 605):
P alleges D sexually harassed her by coming up behind
her and touching her shoulders.
Rule: No evidence of intent to cause harm is necessary
just intent to cause contact

Time, place, and circumstances will necessarily affect its


unpermitted character

Objective Test for Offensiveness: Contact must violate prevailing social


standards of acceptable touching not whether the person touched actually takes
offense
3. Intent (614-22 (through n. 8))
a. Desire or belief with substantial certainty (dont need malice or know its
wrong)
b. Must be volitional (unconscious acts are not intentional)
c. Intent is present where R3 1 (R2 2A)
i. The person acts with the purpose of producing the consequence or
ii. The person acts knowing that the consequence is substantially certain to
result (dependent on facts regarding the knowledge)
Not enough to know that there is a risk; must be certain or near
certain
Foreseeing IS NOT knowing
Much higher threshold for intent than foreseeability
d. Vosburg v. Putney(p. 614:1891): intent to cause contact is all intent you need
i. Context matters. Kicking on playground after recess vs. during
Eggshell plaintiff rule: D is liable for the full extent of damages,
even if the extent is unforeseeable
If D knows about the sensitivity then ?
e. Cole v. Hibberd (1994): Kicked by friend and suffers broken bone
i. this was a battery, therefore loses (because suit is time-barred)
ii. reasonable minds can only conclude that [] intended to kick [] and
that the kick would be considered offensive to a reasonable sense of
personal dignity
4. Single v. Dual Intent (632-33 (nn. 2&3):
a. R3: Intent to contact standard = singe intent
b. Dual Intent: requires the defendant to intend both to make contact with another
and to cause offense or harm by making contact
5. Transferred Intent
a. different tort same victim: One who intends a battery is liable for that battery
when he unexpectedly hits a stranger instead of the intended victim
b. By intending to harm one person responsible if you harm others
i. Restatement Second 16(2)
c. It is not necessary that the actor know or have reason even to suspect that the
other is in the vicinity of the third person whom the actor intends to affect

d. Different victim, different intentional tort/different victim, same intentional


tort: In re White (p. 648, 1982) White tries to shoot Tipton as he flees on his
motorcycle, and accidentally shoots Davis, an innocent bystander
i. White committed the wrongful act when he shot at Tipton.
ii. The injury is not required to be directed against the victim, but includes
any entity other than the intended victim
e. Same victim different tort: Nelson v. Carroll-(p. 651, 1999)- Carroll drew
back the gun to hit Nelson with it, and the gun went off and hit Nelson in the
stomach.
i. Dont need to intend the particular contact
f. From things to persons
i. Lynn v. Burnette (2000): intent to harm chattel=battery?
Court rejects this idea here
6. Knowledge (634-37 (omit Section 3.5):
a. Garrett v. Dailey (p. 634, 1955) five-year-old boy pulled chair out from under
his aunt.
iii. Holding: even if D did not mean to hurt woman, he still had intent if he
knew with substantial certainty it would cause her to fall (knowledge to
a near certainty)
Need more than just a volitional act!
iv. Intent to make contact is probably sufficient
b. Knowledge is a subjective standard in intentional torts
c. Foreseeing is not knowing
d. Knowledge is sufficient not necessary, can still show intent to cause touching
B. Assault
1. Definition: to intentionally cause fear or apprehension of imminent physical harm
1. A acts
2. intending to cause in P apprehension of
imminent harmful or offensive contact; and
3. As act causes P reasonably to apprehend
such contact
completed tort not an attempted tort
2. Act
a. Threat does not have to be capable of being carried out
b. Beach v. Hancock (p. 639, 1853): during an argument with P, D brought out a
gun and fired it twice. The gun was unloaded, but P did not know it was.
i. Held: that even pointing the gun was assault, even if it hadnt been fired

ii. Right to live in society without being put in fear of personal harm but it
has to be a reasonable fear
3. Intent: same as battery
a. Single intent: intent to engage in the act
b. Dual intent: intent to act and cause reasonable contact
i. Subjective and objective elements
ii. Jurisdictions with DI apply it across the board
4. Fear OR Apprehension of imminent physical harm
a. Objective reasonable person standard
i. Social norms and present ability play a role here (i.e. in Booker the Ds
physical proximity made a difference)
ii. R3: subjective standard for apprehension
Says aligns best with interests protected by Assault
i.e. if rock is thrown by A at B, but to an outside observer would
never hit B, nonetheless B is unusually timid and fears he is going
to strike him, can A be held liable?
b. Words are not enough on their own
c. Can make out a PF case without fear, just apprehension
d. Brooker v. Sileverthorne (640, 1919): P was phone operation, and when she
could not connect someone, D said, if I were there, I would break your
goddamn neck
i. Objective standard:
A vain or idle threat does not suffice
Threat has to be of such a nature and made under such
circumstances as to affect the mind of a person of ordinary reason
and firmness, so as to influence his conduct,
OR it must be clear that the person threatened is particularly
vulnerable.
ii. Not reasonable fear in this case because
Lack of proximity
Conditional threat (if I were there
Stated when otherwise angry
e. Conditional threats: not assault unless condition deprives person of legal right
(e.g. If you dont sleep with me)
f. Vetter v. Morgan (P. 643, 1995): Car pulled up next to P at 1:30 am, and men
made threatening and obscene gestures and spat on her van. P swerved to avoid
Ds car and was injured.
i. For words to count as assault, need threat + menace of bodily harm
ii. Social norms play a role in determining

g. Aiding and abetting


i. Encouraging, inciting, or helping carry out the act can make you liable
ii. Similar to concert of action
iii. Phelps v. Bross(p. 647, 2002): Beer girl drugged and raped
Friend with them could be found to have assaulted P by getting
into bed next to her and causing her to fear imminent harmful or
offesnsive contact when she awoke
iv. Alien Tort Statute: Grants federal courts the authority to adjudicate suits
by non-US citizens seeking redress for certain torts defined
independently by the ATS
i.e. Myanmar citizens who were raped and killed by security forces
of US
i. has been limited by SCOTUS, does not permit claims if
occurred exclusively or primarily outside the US or
territories

C. DEFENSES
1. Standard Affirmative Defenses
a. Tend to be justifications not excuses
i. Ds conduct meets the definition of a tort, but all things considered, was
not wrongful (self-defense)
b. Burden on D
c. Common affirmative defenses
i. Consent
ii. Self-Defense & Defense of Others
iii. Defense & Recapture of Property
iv. s fault not a defense (Comparative Fault)
2. Consent
a. All courts agree consent to intentional bodily contact can affect battery liability,
but disagree where within the analysis of a battery claim it should figure
i. Some say element of Ps PFC, P must prove non-consent to prevail (only
a handful of jurisdictions do this)
ii. Others treat consent as an affirmative defense to be pleaded and proved
by D
Stance taken by our book
iii. Majority fails to take a clear stance
b. Implied: based on Ps acts and circumstances
i. Objective reasonable person standard

Subjective belief of P not the test


Consent derived from a source other than the P will not suffice
c. Express: i.e. when someone signs a document for surgery
i. Scope of consent the big question here
d. Consent may be express or implied based on manifest, not actual consent
(what would the other person perceive?)
i. Implied consent defense applies even if D mistakenly inferred consent
ii. Contact sports look to rules of the game
iii. Koffman v. Garnett (p. 656, 2003): middle school football player tackled
by his coach.
[t]aking part in a game manifests a willingness to submit t such
bodily contacts or restrictions of liberty as are permitted by its
rules or usages. R.2d. but doesnt imply consent to contacts
forbidden by the rules of the game if the rules are to protect
participants and not just to make the game a better test of skill
By joining the football team P consented to physical contact with
players of like age and experience, not to aggressive contact by
adult coaches
e. Coercion and Fraud
i. Tortfeasor cannot benefit from consent if he secures Ps consent by
misrepresentation or other forms of deceit
ii. Consent secured through coercion will not count
iii. Also not a defense if:
Victim lacks ability or judgment necessary to give meaningful
consent
A reasonable person would perceive this lack of capacity
i. i.e. minor or mentally challenged
f. Illegal activities: cant consent unless law partly to protect consenting person
i. I.e. Unlicensed boxing match
Majority rule: each party engaged in unlawful brawl is civilly
liable to the other
Minority rule: engaging in a fight is consent to injury, barring
malicious intent
g. Medical privilege: if no family is available to consent, may extend in same
area as long as a function is not lost
h. Emergency doctrine: may treat unconscious patient if the reasonable person
would consent
3. Self Defense: D making an effort to deter a perceived threat
a. Requirements

b.

c.

d.

e.

i. Only if actually and reasonably perceives imminent risk of physical


injury
ii. Reasonable belief the use of force is necessary to repel an attack or
imprisonment
iii. The use of force must be proportional to the perceived threat
Can only use deadly force to repel a serious or fatal attack; might
be > is necessary (water gun)
iv. Does not apply:
When conduct threatens only to result in defamation or distress
Does privilege force to respond to nonthreatening provocations
Purposes
i. Protecting personal dignity
ii. Deterrence
iii. In accordance with human nature
iv. Conflict with goal of torts to supplant private methods of redress for
harm
v. Prevent attempted murderers / rapists from being able to sue their victims
Availability: When a victim who actually and reasonably believe it is necessary
to injure another to avoid imminent injuries to herself such as harmful contact
i. Hauessler v. De Loretto (P. 666, 1952): P went to neighbor's house to
look for dog and got angry, D asked P to leave and then struck him once;
knew of Ps past violent behavior
D acted in self-defense and used reasonable force...
Retreat: courts split on duty to retreat, but second restatement says
non-deadly force is ok, but deadly force not, in lieu of retreating
Privilege of causing harm that would otherwise be tortious
i. Reasonable force (not deadly) when battery is anticipated. No duty to
retreat (R2 63)
ii. Can use deadly force when like is anticipated; duty to retreat (R2 65);
castle exception
iii. Exception of Castle Doctrine (common approach?) in Florida, there is
a presumption that when someone enters your home, they are entering to
harm you and you may take deadly measure
Deadly force: Only justified when injurer actually and reasonably perceives
that the victim is threatening him with imminent death or serious bodily injury
i. Rule of retreat R265(3): If actually believe one can safely retreat from a
confrontation that would otherwise justify the use of deadly force, one is
not justified in using such force

Does not apply to the use of non-deadly force or to the use of


deadly force to ward off an intruders attack in ones dwelling
f. Stand Your Ground: Expand privilege to use deadly force
i. Zone/Space 1: occupied house or vehicle
If another forcibly enters, Ds reasonable apprehension of
imminent serious bodily harm is presumed (irrebuttable)
ii. Zone/Space 2: anywhere else one has a right to be
No duty to retreat (even if easy and safe)
When D provokes another to attack him, D faced liability for
injuries caused by use of defensive force unless
i. The other continues altercation after D breaks off; or
ii. During the engagement, D has no option but to use force to
save himself from serious harm (i.e. retreat is required if
available)
g. Defense of others: operates under same reasonableness/imminent harm
standard
h. Defense and Recapture of Property
i. May repossess goods as long as does not involve a breach of peace
Exercised at ones own peril
ii. Katko v. Briney (P. 670, 1971): Spring gun in a barn injures trespassers
looking for glass jars
The value of human life and limb both to an individual and as a
matter of public policy outweighs the potential damage to property
Rule: While a D may use reasonable force in defense of her
property, he has no right to willfully and intentionally injure a
trespasser in a manner that may result in loss of life or great bodily
injury
i. Exception: trespasser is committing a violent felony with the
potential of endangering human life
- Unclean Hands (Kato Dissent): a party who is asking for a
judgment cannot have the help of the court if he/she has done
anything unethical in relation to the subject of the lawsuit.
i. Applied by courts faced with requests for injunctive
relief as opposed to damages
D. FALSE IMRPISONMENT
1. Definition: Intentional Confinement
a. A acts
b. Intending to confine B

c. As act causes B to be confined and


d. B is aware of his confinement
-Some Jurisdictions add (but most treat these as affirmative defenses)
e.
P did not consent to be confined; and
f.
A was not legally authorized to confine P
- Type I: confines under claim of authority (police arrest, detention by store
guard)
i. Key issue: authorization
- Type II: confines w/o pretense of authority (adult friend locks friend in closet)
ii. Key issue: elements, consent
- Fojtik v. Charter Med. Corp (P. 677, 1999): Man sent to rehab against his will
- To prevail on a claim of false imprisonment through the use of threats despite
never insisting on release, the facts must show a vulnerable plaintiff confronted
with an oppressive, intimidating environment.
2. Confinement: occurs when the tortfeasor causes the victim to be within a bounded
physical space
a. Can be small or large space, but free movement does not count
b. Does not need to be stationary
c. Can occured even if there is an exit, but exit can only be used in a manner that
risks physical harm
d. Emotional fear/mental confinement does not count
3. Awareness: Requires apprehension of the condition of being confined
a. R2(35) require the P to prove that she was aware of her confinement as it
happened
4. Shopkeepers privilege(Defense) authorize someone with reasonable suspicion of a
crime to take reasonable measures to detain someone for purposes to determine
whether that crime has occurred
a. Limited number of potential defendants have this privilege
b. can detain a person in a reasonable manner for a reasonable time
c. test: is general not based on the store patrons guilt or innocence, but on the
reasonableness of the stores action
i. usually determined by the trier of fact
5. Grant v. Stop-N-Go Market of Texas, Inc. (P. 685, 1999): A plaintiff may sustain a
claim for false imprisonment if the plaintiff was willfully detained by threat, without
any physical restraint.
a. Alters common law rule of detain at your own peril
NEGLIGENCE

A. Overview and Elements: A breach of a duty to avoid causing injury to another through
careless conduct
1. Injury
a. Physical damage, property damage (general, unqualified)
b. economic loss, emotional (limited, qualified)
2. Duty
3. Breach of Duty
4. Causation (Actual and Proximate)
B. Background:
1. Principal common law writs
a. Medieval England usage Trespass had a very broad meaning
b. Trespass vi et armis (with force and arms)
c. Trespass-on-the-case
i. Absent act of aggression
2. Brown v. Kendall (p. 806, 1850) D, breaking up a dog fight, inadvertently hit P in
the eye with a stick
a. Liability requires fault not strict liability
b. Liability under the old writ of trespass arguably was strict as to forcible and
directly caused injuries
c. D cannot be liable for the results of lawful and proper act taken under ordinary
care
d. Merged the two writs and required negligence for both, shifted burden to P
e. Establishment that accident based tort liability is and has always been faultbased rather than strict
f. Ordinary Care Standard: an accident victim cannot recover from a person
who played a role in bringing about the accident unless the person caused the
accident by failing to exercise ordinary care.
3. Harvey v. Dunlop: Fault or carelessness is required before tort liability will be
imposed on an actor who accidentally injures another
4. Holmes: The undertaking to redistribute losses simply on the ground that they
resulted from the defendants act would . . . offen[d] the sense of justice. Unless my
act is of a nature to threaten others, unless under the circumstances a prudent man
would have foreseen the possibility of harm
A. Duty
1. Definition: requires a negligence plaintiff to establish that the defendant owed her, or
a class of persons including her, an obligation to take care not to cause the type of
injury that she suffered
negative duty-duty to take care to avoid causing harms

positive/affirmative duty-duties to make reasonable efforts to protect or rescue


another
a. Easy duty cases malfeasance. Affirmative course of conduct that caused
physical harm
b. Hard duty cases failure to act or non-physical harm
c. Modern standard a person owes a duty of care not to cause physical harm to
those around them when it is reasonably foreseeable that other persons might
suffer physical harm as a result of that conduct
d. Duty is a question of law for the judge
e. Underlying rationale = uniformity, putting parties on notice
i. Reasons for limitation liability only where there is a duty
ii. Tradeoff between accountability and notice
iii. Foreseeability unforeseeable harms cannot be deterred
iv. Autonomy protect actors freedom of choice
v. Corrective justice redress acts that are blameworthy
vi. Administrative ease tort system is less costly, more efficient
f. CA SCt: presumptive duty where injury is foreceable (MacPhersons general
duty rule for ALL negligence cases, including nonfeasance), but courts can
grant no-duty exceptions for policy reasons
g. Majority Rule: presumptive no duty for nonfeasance w/ special yes-duty
exceptions
2. Evolution of Duty Rules
a. The privity rule required contractual relationship for there to be duty
b. Winterbottom v. Wright (P. 56, 1842): privity rule established requiring a
contractual relationship for there to be duty
c. Thomas v. Winchester (P. 57, 1852): Imminently dangerous products became an
exception if death or grave bodily harm is the natural and almost inevitable
consequence of the sale of the product, then duty extends to those not in privity
with the manufacturer
d. Heaven v. Pender (1883): duty to P if D reasonably should foresee that
carelessness would risk injury to a person such as P
e. No Duty Decisions:
i. Loop: wheel had been used w/o incident for 5 years, then leased
ii. Losee: seller knew buyer would test
3. Easy Duty: cases in which neither the litigants nor the court will spend much time
on the issue because all are satisfied that the person being sued owed it to the
complainant to take reasonable care
a. Macpherson v. Buick Motor Co. (P. 60, 1916): D sells car to dealer, dealer sells
car to P, Ps car collapsed.
i. Rule (Cardozo):

a. Narrow reading: If a product, when carelessly made, is reasonably


certain to place life & limb in peril, and if the manufacturer knows
the product will be used by non0purchasers, and not inspected fir
safety after sale, the manufacturer owes a duty to product users to
make it carefully
b. Broad reading: manufacturer owes a duty to anyone whom it can
reasonably foresee injuring by its carelessly made product
c.
ii. Important rejection of the notion that freedom of contract is
paramount among legal principle governing private interaction
a. Historical significance recognizing changing economic political
realities
b. Mussivand v. David (p. 68, 1989): Doctors wife slept with another Doctor and
then infected Dr. Mussivand with a venereal disease.
i. Duty to abstain or at a minimum warn
ii. Existence of duty depends on foreseeability of injury.
iii. Qualifies and says what duty entails, not just to whom it is owed
iv. Relaxes McPherson: meaningful possibility
a. need not be a large statistical probability (need not be > 50%)

4.

c. Foreseeability: any person whom one can reasonably foresee physically


harming if one were to act carelessly
i. Shield: if one can anticipate that ones acts might injure certain others,
one is morally obligated to those persons to take reasonable steps to
avoid injuring them
ii. Sword: it wouldnt be fair for law to impose a duty and liability for
failures to take steps against causing injuries that one could not have
anticipated
d. R3: Focus on relationships, says foreseeability should be irrelevant
i. Slippery slope issue: has been criticized by many saying its asking
judges to make these decisions. Let the cases go to the jury.
e. Special type of normative/moral foreseeability
i. Not eggshell foreseeability/harm foreseeability
ii. More objective
iii. Works at fairly general level
Qualified/Limited Duties of Care
a. Rule: no duty to rescue generally
i. Rationale:
a. Too much liability on defendants
b. Floodgate concerns (cost of administration

b.
c.
d.
e.
f.
g.

h.

i.

j.

c. Undermining private insurance incentives


d. Discouraging defendants or others from taking precautions
e. Fairness come into play through foreseeability (proximate cause)
Affirmative duty or duty to rescue cases: unreasonably failed to provide
assistance or protection to the plaintiff
Premises liability cases: permitted or maintained unreasonably dangerous
conditions on property in his or her possession
Pure Economic Loss Cases: Acted without reasonable care for the plaintiffs
economic prospects
no duty: applies when Ps complaint is that D failed to render assistance in
which assistance could have prevented the Ps injury
i. nonfeasance
Misfeasance v. nonfeasance
i. If action (misfeasance) duty [of reasonable care]
ii. If omission (nonfeasance), no duty unless special relationship
Osterlind v. Hill (p. 77, 1928): A lessor of a boat has no legal duty to refrain
from renting to, and consequently to rescuing, an intoxicated person who is not
clearly helpless.
i. No duty to aid even when it is safe and easy
ii. Possible argument that there was a duty to rescue even if we accept
that this is a case of nonfeasance and not misfeasance when D can be
seen as having helped to create the risk, D has a duty to make a
reasonable effort to prevent victim from suffering further harm, or the
risk from being realized.
iii. This is a stronger exception than the pre-tort special relationship, and
probably why Osterlind would come out differently today
Baker v. Fenneman (p. 78, 2003): Taco Bell had a duty, as a business that
invited members of the public to enter its facility to provide reasonable
assistance to Baker even though Taco Bell was not responsible for Bakers
illness
i. Key difference between Osterlind is special relationship
a. Degree of control over the potential risks is crucial
Duty analysis under Fenneman
i. Relationship between parties (store owners invited person on land)
ii. Reasonable foreseeability of harm to the person injured
iii. Public policy concerns (businesses should be doing this anyway)
R2 314A Special Relationships Giving Rise to Duty to Aid or Protect:
a. Factors that appear to distinguish these relationships
i. Custody
ii. Knowledge of risk

iii. Degree of control


1. A common carrier is under a duty to its passengers to take reasonable
action:
a. to protect them against unreasonable risk of physical harm,
b. and to give them first aid after it knows or has reason to know that
they are ill or injured, and to care for them until they can be cared for
by others.
2. An innkeeper is under a similar duty to his guests.
3. A possessor of land who holds it open to the public is under a similar
duty to members of the public who enter in response to his invitation.
4.One who is required by law to take or who voluntarily takes the
custody of another under circumstances such as to deprive the other
of his normal opportunities for protection is under a similar duty to the
other.
-Not an exclusive list
-Only applies while relationship exists
k. Exceptions to no duty:
i. Duty to rescue statutes
a. Vermonts Duty to Aid the Endangered Act: imposes a duty of
reasonable assistance on any person who knows that another faces
grave physical danger, if assistance can be rendered without peril
to the rescuer. Violations punishable by fine.
ii. Imperilment Imminent Peril to P caused by D (common law)
a. Actor knows or should know he has by his own conduct caused the
victim to be physically injured and at imminent risk of further
injury
b. Actor has a duty to make reasonable efforts to prevent the victim
from suffering further harm or to prevent the risk of harm from
being realized
i. i.e. A buys drugs for B and B overdoses
ii. friend beat up, put him in back of car, in the morning in a
coma (beginning to come to aid but not following through)
iii. Voluntary Undertakings (common law)
a. D has volunteered to protect another from physical injury or
property damage, or to rescue another from physical peril
i. May arise from contractual promise or less formal
undertaking
b. Related exception: once a rescue is voluntarily undertaken, the
rescuer owes a duty to the victim to take reasonable care in the
rescue

iv. Special Relationships (see above)


a. Carrier-passenger
b. Landowner-guest
c. School-student
d. Employer-employee
e. Hospital-patient
f. Prison-prisoner
g. Business invitee
l. Good Samaritan Immunities: immunize voluntary rescuers from tort liability
for negligence
i. Enacted by every state
ii. Tend to be limited to off duty professionals like Doctors

5. Premises Liability: obligations for possessors of land


a. Leffler v. Sharp (p. 90, 2004): Bar patron injured when climbing through a
window to patio that was off limits
i. Defines categories
ii. Shows how status can change
iii. Does not mean liable just that will go to jury
Plaintiffs Status
Ds Duty to P
invitee: enters at invitation of possessor in
must take care to provide reasonably sa
furtherance of possessors business or mission premises
licensee: enters w/permission of possessor
but not in furtherance of possessors business
or mission

must warn of hidden dangers about whi


the possessor knows (or should know)

trespasser: enters without permission

-no duty of care owed to adults (excepti


for known trespassers)
-duty owed to children not to maintain
attractive nuisance, or 4cbly dangerous
condition

b. Ownership: not required to be a possessor for premise liability claims


c. Hazardous conditions v. Hazardous activities
i. Designed to address things like holes, wires, not activities
d. Trespassers: vast majority of states continue to adhere to no-duty owed to
trespassers, only a duty to refrain from willful or wantonly injuring them
i. Exceptions

a. Habitual trespassers: if the possessor knows or that trespassers


constantly intrude upon a limited portion of the premises so as to
be exposed to the danger
i. i.e. property with a path that is commonly used as shortcut
b. Attractive Nuisance: obligation to not maintain a condition that
would pique a childs curiosity but that contained a danger that
would not be apparent to such a child
i. i.e. pond or pool
e. Roland v. Christian (p. 115, 1968):Always a duty owed by a property possessor
to anyone who comes onto her land, no matter what the circumstances of the
entry
i. Common law standards are arbitrary and shouldnt value human life
differently
ii. Eliminates categories
iii. Eliminates doctrines that allow judges to take cases away from juries
iv. Judges should never dismiss premises liability claims on no-duty
grounds
v. Transfer of power from judge to jury
vi. Fewer than 10 states follow this approach
f. R3: favors variation on Rowland
i. General duty to take reasonable care to make premises safe for all
persons including adult trespassers
ii. Flagrant trespassers (those committing serious crimes /felonies): only
duty is from intentionally, willfully, or wantonly causing physical
harm

Again, does not denote liability, just that will go to the jury

6. Professional Duties
a. Tarasoff v. The Regents of CA (p. 125, 1976): 1976)--Psychiatrist D treats
patient who said he was going to kill decedent P; psychiatrist tells police to
restrain patient which they do but they promptly release him, and no one ever
tells P or her family; she is killed upon her return from abroad.
i. Duty to warn or take reasonable steps, particularly based on
professional standards, since doctor knew or should have known of
credible threat to 3rd party.
ii. Need not be actually aware patient is contemplating attack, so long
as therapist reasonably should have known of the risk
iii. Court is clear this is not misfeasance
iv. Only an obligation to warn, not requiring perfect performance
v. Victim must be identifiable
vi. Aspects of Special Relationship:
a. Control
b. Knowledge
c. Custody
vii.
Favoring Duty concerns:
a. Protection of would-be victims
b. Manageable for MDs (only a duty to take steps to warn)
c. Less liberty-infringing for patients than confinement
viii.
Against Duty
a. Patients interest in confidentiality
b. Societys interest in not discouraging therapy
c. Ineffectual or generative of too many warnings (amici curae)
b. Scope of duty
i. Courts split on what I required for duty to attach
ii. CA recognizes person must be identifiable
c. Causation issues caused by Tarasoff
i. How do we know a warning would have worked?
a. Get to trial, jury to decide info would have made a difference
d. Declined to extend Tarasoff to clergy
i. Lack of training
ii. Policy reason of not intervening in the church
e. Duties to protect Third Parties from Physical Lapses
i. Some courts reject aTrasoff majority that says physician under duty to
warn patients with disorders such as epilepsy they may suddenly
become incapacitated while driving
f. Calebresi and the Cheapest Cost Avoider

i. Tort law should aspire to assign liability to the cheapest cost avoider:
the person or entity who can identify and adopt the most efficient
precaution more readily than anyone else
7. Pure Economic Loss: (no physical injury or personal property damage)
a. Can recover for economic loss when attached to any claims of property
damages or injury
i. Ex: when an explosion causes economic loss in an area, businesses hit
with debris can recover for damages and economic loss; businesses
not hit can recover for NOTHING
ii. Ignores misfesiance/nonfesiance focus on physical harm
b. Policy for no recover from PEL
i. Theres no net loss for society, just a transfer of where money is spent.
ii. Indeterminate and potentially limitless liability
iii. Businesses should buy own business interruption insurance (cheaper
for society overall, no litigation costs, least cost avoider); encouraging
this
iv. Law doesnt protect peoples wealth or right to generate income, alone
(no property right to have a business and no liability of other party for
putting company out of business)
v. Proof issue: maybe it was something other than this wrong that caused
economic loss (economy dropping, etclike in BP)
vi. How know that company wont/didnt regain sales upon re-opening?
c. Policy Against recovery for PEL
i. Under-deterring wrongdoer?
ii. Justice! Innocents shouldnt have to pay!
iii. Fortuitousness of businesses that suffer property damage too
(arbitrary)
iv. If law doesnt protect businesses, then why not let business doing the
harm suffer the consequences?!
d. Aikens v. Debow (P. 101, 2000)--Truck driver damages highway forcing closure
impeding business of P motel owner; hotel sues trucker for pure economic loss
i. P cannot recover because trucker had no special relationship or
knowledge leading to foreseeability of economic loss; despite
highways proximity to motel and existence as main entry (other ways
to get to motel)
ii. Rejects NJ minority approach: Peoples Express (p. 111-12, 1985)
Ds negligence caused a dangerous chemical to escape from a railway
tank car, resulting P being forced to evacuate its premises and suffer
interruption of business

e.

a. P wins pure economic damages; court establishes test of liability


where if D has special reason and can reasonably foresee
damages =>liable (subjectively foreseeable part of evacuation
plan, permanent physical proximity, argument for property damage
= evacuation, chemical gases infiltrated building)
b. Particularly foreseeable plaintiff rule: D must have knowledge
or special reason to know of the consequence of the tortious
conduct in terms of the persons likely to be victimized and the
nature of the damages likely to be suffered.
iii. WV approach: test is impact
a. Physical contact that could cause injury is enough
b. Doesnt actually require injury
Generally cannot recover for PURE ECONOMIC LOSS, exceptions:
i. Special relationship: accountant doing audit which knew would be
relied upon; (also sometimes attorneys considered special
relationship)
a. (1) Knew WHO would affect (exact identity of creditor known,
quasi-privity relationship between creditor and auditor)
b. (2) Knew HOW would affect (know that work is being relied upon
by a specialized group, just dont know the identities of the
members of the group); OR
c. (3) Generally foreseeable (most liberal, least popular...just know
that is likely to be used by SOME party)
ii. Statutes
a. i.e. Oil pollution control act
i. fault based strict liability
iii. Fishermen: within 3 miles of shore, public trust of all citizens of
state; fishermen ONLY have a quasi-property right to state waters, so
may be able to recover for pure economic loss due to pollution.
There can be a step[s] between property damage and loss of income, but there
must be a chain of causation proving actual loss.
If using property for something like a sport there is a duty to take reasonable
care
Spirit of Chapter 2: Courts will find a duty if they want

B. BREACH
1. Violation of established duty
2. Standard: Average reasonable Person/Person of Ordinary Prudence
a. Objective standard

b. Cant read minds so its easier to make everyone live up to one standard (more
accurate, less admin costs).
c. External conduct not internal attitude of concern
d. Encourages societal baseline (no societies of idiots).
e. Deters fraud in avoidance of negligence (just didnt know, etc.).
f. Incentivizes better judgment.
o Holmes rejects this in favor of reducing the general ordinary prudence standard to
specific rules of conduct
o Ripstein: the employment of a subjective standard would violate an entitlement of each
person to enjoy an equal degree of protection from others' risky conduct
3. Burden of Proof
a. Production: the requirement of providing to the court some evidence in support
of an alleged fact or set of facts that the party bearing the burden seeks to
establish
i. Often circumstantial
b. Persuasion: comes into play only after there is evidence before a judge or jury
upon which it must make a decision
4. Vaughan v. Menlove (p. 166, 1837): D stacked hay that posed a risk of igniting,
warned by P, disregarded warning, hay ignites and damages Ps cottages.
a. Instead of saying that the liability for negligence should be co-extensive with
the judgment of each individual . . . we ought rather to adhere to the rule
which requires [the degree of care that] a man of ordinary prudence would
observe.
b. Rejects argument that D particular sensibilities or weaknesses should be taken
into account
i. Not taken in to account (can apply as comparative fault
determination)
a. Clumsiness
b. Foolishness
c. Mental illness
d. Old age
ii. Taken into account:
a. Youth (Tender years Doctrine)
b. Physical disability (blind person compared to ordinary blind
person)
c. Expertise (higher standard R3)
i. Pro-skier has higher expectations
5. Appelhans v. McFall (p. 168, 2001): D carelessly riding a bike, and hits elderly
woman who breaks her hip, brings suit against child and parents

a. Tender Years Doctrine: children cannot recognize or appreciate risk and are
relieved of liability
i. Illinois (minority approach) (ages 1-7 TYD applies, 7-14 duty to act
prudently as a child of like age and experience, unless engaged in
adult activity
ii. Court recognizes ridiculous nature of 7 year cut off, but reasserts
because stare decises
6. Tender Years Doctrine:
a. Restatement (majority):
i. Child negligent if conduct does NOT conform to that of a reasonably
careful person of the same, age, intelligence and experience
UNLESS:
a. Under 5 yrs old = not capable of negligence
b. Engaging in a dangerous activity that is characteristically
undertaken by adults
b. Massachusetts Rule: child capable of...negligence if child failed to exercise a
degree of care that is reasonable for similarly situated children
c. POLICY reasons for holding young child liable:
i. Parents should pay b/c least cost avoider (but then why not hold
directly liable?)
ii. Old enough to start learning morals (deterrence)
iii. Compensation for victim (STRONGEST rationale)
a. Encourage parents to buy insurance
iv. BUT compensating one victim through money of another innocent
(like strict liability for parents; dont even have to be negligent
parents)
v. Is only being held to standard of like-aged kid, so only found liable if
truly extreme behavior (BUT how determine what is normal?)
vi. BUT really young children CANNOT understand deterrence (though
indirect deterrence through parents educating children on what not to
do)
vii.
BUT dont want to restrict kids lives to the point that they can
do nothing; should explore; is part of childhood.
7. Parental Liability:
a. Person injured by childs carelessness must establish some form of direct
negligence on part of parent to recover
i. Negligent Supervision: If parents know of minors tendency to
commit wrongful acts and also failed to exercise reasonable means to
restrain him, thereby enabling his toritus conduct

ii. Negligent Entrustment: parent carelessly gives a child access to a


dangerous instrumentality, such as a gun or a car, that child is not
equipped to handle safely
8. Reasonable Child and Comparative Fault
a. Generally used to assign percentage or responsibility and liability to the parents
for an injury to their child
9. Mental Incompetence
a. No insanity defense in tort
b. Even if so incapacitated you are incapable of forming the relevant purpose, not
liable for battery, still can be liable for negligence
c. Rationale here:
i. (1) the desirability of allocating losses between two innocent parties to
the one who caused or occasioned the loss;
ii. (2) providing an incentive to family members and other guardians of
persons with mental disabilities to control the behavior of those
persons;
iii. (3) removing inducements for alleged tortfeasors to fake a mental
disability in order to escape liability;
iv. (4) avoiding administrative problems that are created by requiring
courts and juries to identify and assess the significance of an actors
disability; and
v. (5) forcing persons with disabilities to pay for the damage they do if
they are to live active lives
d. Breunig: declined to exempt persons who satisfy clinical definitions of insanity
i. However those who suffer temporary bout, not required to meet
standard
a. i.e. heart attack
ii. Does not protect foreseeable bouts
a. I.e. Epilepsy while driving if you know you are epileptic
10.Gender
a. Should gender be taken into account?
i. Some gender discrimination has used reasonable woman standard
11.Culture
a. Vietnamese case Ha v. Alaska
i. CoA even taking into account knowledge of Vietnamese culture, HA
had no basis for perceiving an imminent threat from Buu at the time
Ha shot him
12.Time Frame for Assessing Conduct
a. Negligence law generally permits P to prevail on breach if she can identify any
time-slice in which conduct fell below reasonable care

i. A doctor with a great history can have one moment and history
doesnt matter
13.Ordinary vs. Professional SoC?
a. Myers v. Heritage Enters (p 144, 2004): Should jury use professional standard
of care of ordinary care standard when injured using Hoyer lift
i. Professional negligence applies to situations of skilled medicare only
and not administrative or other hospital duties
14.Foreseeability
a. Adams v. Bullock (p. 158, 1919)(Cardozo): P injured when swinging a wire and
hit a trolley line below the bridge
i. Negligence should be applied only when the injury is fairly within the
area of ordinary prevision
a. No special danger at this bridge
b. No like accident had occurred before
c. No custom had been disregarded
d. Trolley wire cannot be insulated
15.Standard Jury Breach instruction
a. The jury is instructed that negligence (breach) means a failure to use
ordinary care.
b. Ordinary care is defined by reference to a reasonably careful person or a
reasonably prudent person, and occasionally by reference to a reasonable
person.
c. The instruction directs that the jury is to consider whether the defendant was
negligent in doing something OR in failing to do somethingthe jury is to think
both about what a defendant has done and about what the defendant has not
done.
d. The jury is required to consider the circumstances in which the defendant acted,
and whether the defendant acted with ordinary care given the circumstances.
16.Duty and Breach
a. Aikens: breach but no duty
b. Martin v. Evans: duty but issue over breach
17.Extrodinary Care
a. Common Carriers often held to greater-than-ordinary care
b. Many modern courts say use ordinary reasonable person
i. Bethel: since ordinary reasonable person takes into account all
relevant factors, no need for heightened standard
18.Recklessness
a. To prove recklessness P must prove there was a high probability that a person
would be seriously injured or killed
19.Emergencies

a. Courts split here


b. All agree jury is entitled to take it into account
c. Split over whether to alter instruction specifically addressing the emergency
i. i.e. Katrina
20.Holmes (Goodman) v. Cardozo (Pokoro)
a. Holmes: judges should more frequently issues JMOL on breach
i. Stop and look standard
b. Pokoro: abandoned Goodmans rule
i. Need for caution in framing standards of behavior that amount to rules
of law particularly where there is no background of experience out of
which the standards have merged and are artificially developed
ii. Where ordinary safeguards fail judgment for jury
21.Custom (fact finding up to jury)
a. R3 13 General rule: compliance or non-compliance is admissible, but not
dispositive. Jury allowed to treat it as a standard of care or just evidence
(usually not the standard).
b. Exception: medical malpractice (custom is the standard usually)
c. Custom as standard of care in med mal (also below for Lack of Informed
Consent Professional Standard):
i. Pros
Consensual relationship between doctor and patient
Reduces litigation cost
Predictability ex ante
Doctors are held accountable by their peers and their own society,
as well as tort law
Juries cannot create standards of care for medical malpractice
casesits too technical
ii. Cons
Insurance companies are stepping between doctor patient
relationship and disincentivizing doctors from doing everything to
save patient
Doctors are protecting their own
Costs get externalized to 3rd party insurer making standard of care
too high or too low
Does patient have equal bargaining power?
Is there actually a custom?
i. doctors take into account non-medical factors (psychological
factors etc)

d. The TJ Hooper (p. 178, 1932): D operated a tugboat without radios, as was the
custom. The tugboat sank in a storm; cargo owner sued, alleging that loss would
have been prevented with radios
i. D was still liable even though he followed industry custom
ii. Rule: Compliance with custom is probative of ordinary care but not
dispositive
e. Johnson v. Riverdale Anesthesia (p.182, 2002)-- patient died from anesthesia
because she was not pre-oxygenated, P tries to impeach Ds expert witness by
asking what he would personally do in similar situation, though he stated
industry custom agreed with D;
i. court found that professional standard is not about what an individual
doctor would do, but rather what the society of doctors would do
ii. Anti TJ Hopper Rule: AMOL, no breach when D adheres to
professional custom
f. Condra v. Atlantic Medical Group (p. 185, 2009): Expert witnesses for D
introduced alternate opinions regarding treatment
i. Testimony on how they would have treated P is relevant
ii. Overrules Johnson
Sometimes customs may be modified, need to let new facts in
iii. P friendly, but under Johnson P might not even need to hire a witness
g. Minority rule- if a respectable minority of doctors are following a rule then
thats enough for custom
h. Locality rule: look at geographic standard of care
i. problem if a small town
ii. problem if we rely on area where doctors are colleagues
i. R3 treats conformity to custom and departure from custom symmetrically; both
may be used as evidence by the factfinder to ascertain whether there was a
breach of duty
j. Courts typically maintain that a practice must be "widespread
or "common" to count as a custom
k. In medical cases proof of compliance with professional custom often does
establish reasonable care as a doctor is required to exercise the same level of
care as is considered standard by members of the profession
l. Johnson notes unless the professional conduct at issue is so unrelated to
professional expertise that lay jurors can assess it based on their experience, the
plaintiff in a malpractice action must introduce expert testimony to establish
that the D failed to heed standard of care
22.Informed consent admits exceptions
a. No informed consent need be obtained to operate on an unconscious patient in
need of immediate surgery

23.Largey v Rothman (P. 188, 1988): P had mass in her breast and had to have a biopsy,
which led to arm problems that were rare but foreseeable by the doctor; doctor had not
warned of this potential risk; P says would not have had procedure if had known
NJ SCT
a. .MDs must provide info. that a rsble patient in s circumstances would deem
material to her decision
i. for informed-consent malpractice, return to the T.J. Hooper Rule
(custom probative, not dispositive)
b. Causation determined by objective standard: would a prudent patient in s
circumstances have declined the procedure given proper disclosure of the risks?
i. would a subjective standard be more consistent with concern for
patient self-determination?
c. Lack of informed consent
a. Prudent patient standard (Largey - below)-- a physician should
disclose material risks that a prudent/reasonable patient would
want to know (patients rights of self determination - autonomy)
b. Professional standard -- doctor should inform patients of what a
doctor would generally have said (doctor in the community v.
comparable communities v. national networks)
ii. Elements of a lack of informed consent case (duty assumed):
a. 1) Standard of care breached (one of the following):
i. Prudent patient standard
ii. Professional standard
b. 2) Proof injury occurred at all
c. 3) Causation-- injury was caused by the risks that were not
disclosed to P by doctor
i. Objective approach-NO reasonable patient would have
accepted the treatment if informed
ii. Subjective approach-- the patient personally would NOT
have accepted the treatment if informed
d. Should we switch to prudent patient standard?
i. Pros:
a. Autonomy theory (avoid paternalism)
b. Difficulty of patient getting experts (doctors protect own)
ii. Cons
a. Juries will still resort back to professional custom even if
instructed on prudent patient custom
b. Competition amongst doctors could cause high costs

c. Not always in patients best interest (dont want to terrify them)


d. Incentivizes over-treating patient
iii.
24.Per Se rule extends to other professions including legal and accounting professions
25.Cost-Benefit Analysis and Reasonableness
a. US v. Carroll Towing(p. 200 1932)(HAND) : tug causes barge to hit tanker and
sink, accident could have been prevented by having 24-hour a day bargee, but
bargee was not present at time of accident
i. Admiralty laws pro rata rule for cases of property damage:
a. an actor whose fault contributes to the damage is responsible for
the $-value of the damage divided by the number of actors whose
fault contributed to the damage
ii. Hand formula: B = P x L
a. Ex Ante perspective (before accident not in hind site
b. B = burden on D to take the precaution
c. P = probability of injury
i. BUT probability that the precaution wont prevent the
injury? (Would people really use helmets if you put them in
every car?)
d. L = gravity/severity of injury
e. If B > P x L then should NOT take the precaution; D is NOT liable
f. If B < P x L then should take the precaution; D is liable.
b. Policy considerations of Hand formula:
i. Advantages
a. Predictability, maximize social utility, simplicity
b. Guide judges on appeal
c. Acknowledges that precision is illusory but it centers attention on
relevant considerations
d. Cost internalizing pro plaintiff, influences conduct of lawyer
i. Untaken precaution becomes legally relevant
e. Technology forcing pro plaintiff since B put into evidence
f. Adaptive pro defendant
g.
ii. Disadvantages
a. Promotes societal inequalities (better to speed in a poor
neighborhood than in a rich one)
b. Not widely used b/c juries normally determine reasonable standard
of care
c. Information intensive-- must monetize everything

i. Therefore, actually less predictable.


ii. Administrative costs
d. Subjective = how to value the benefits/costs (Keating and moral
values)
e. Statistics are rarely available
iii. Alternatives: community expectations, virtue, substantial risk
iv. Highlights Ps burden to point to facts of feasible burdens
26.Posner on Hand
a. Sets basis for dollar cost (economic application) dollar cost v dollar value
b. Utilitarianisms approach, social arrangements should be to maximize the
aggregate level of utility or happiness in the world
c. All about waste
d. McCarty(POSNER)- upholds D verdict where woman gets attacked because
landlord negligence
27.Lord Reids Disproportionate Cost Test
a. If at the time of acting the risk that actors conduct would cause harm to P was
exceedingly small, no obligation to take precautions against it
b. If risk of harm was not far-fetched real but still very small, actor is obligated
to take precatuions, unless would have imposed a burden on the actor entirely
disproportionate to the harm risked
c. If risk of harm is material or substantial, actor obligated to do everything
possible to prevent the harm even if that requires expensive precautions to
avoid a modest expected loss [differs from hand, standard is [utmost care]
d. R3: reasonable person standard to embrace balancing
i. Bolton v. Stone P hit by a cricket ball from adjacent field
a. Lord Reid offers an alternative rights-based approach that does not
take into account the burden of precautions.
b. Disadvantages imposes burdensome precautions
c. Reids approach not adopted anywhere, so dont use in analysis
A person acts negligently if the person does not exercise reasonable care
under all the circumstances. Primary factors to consider in ascertaining whether the
person's conduct lacks reasonable care are:
- the foreseeable likelihood the person's conduct will result in harm,
-the foreseeable severity of any harm that may ensue,
-and the burden of precautions to eliminate or reduce the risk of harm
28.Res Ipsa Loquitor (the thing speaks for itself):
a. Allows P to prevail on breach without producing evidence as to the precise way
in which D was careless

b. Allows P to establish PF case regarding breach


c. If judge decides P can use RIL D cant get case dismissed even though no
evidence
d. If jury finds D evidence convincing can still find for D
e. Merely permits permissive inference. Judge and jury both have role
f. Elements:
i. Injury must be a kind that ordinarily does not result absent
carelessness on someone's part
ii. Instrumentality causing the injury must have been in D's exclusive
control
iii. Injury must NOT have arisen from acts or carelessness on the part
of the P
a. Duty still must be established
b. Focus generally on the first two elements
g. Byrne v Boadle (P 216, 1863): P struck by barrel of flower and brings case
against D
i. Rule: A plaintiff must persuade a jury that more likely than not the
harm-causing event does not occur in the absence of negligence.
a. The plaintiff does not have to eliminate all other possible causes
for the harm, nor does the fact that the defendant raises possible
non-negligent causes for the harm defeat plaintiffs effort to invoke
res ipsa loquitur.
b. The key is that a reasonable jury must be able to find that the
likely cause was negligence.
h. Kambat v. St Francis Hosp. (P 218, 1997): patient got a hysterectomy; doctors
left a pad in her
i. P does NOT have to eliminate all other possible causes of injury to
use RIL; P simply must show that it is more likely than not that Ds
negligence caused injury.
ii. DONT need an expert in RIL med mal cases b/c so obvious (though
most med mal cases usually use experts - tension)
a. Most courts follow this idea if injury is so glaring RIL is
appropriate, jury confusion not an issue
i. First element
i. Often bars RIL in the vast majority of negligence
a. Medical procedures often produce complications and injuries even
without negligence
j. States v. Lourdes Hospital (2003): RIL can be invoked even if expert testimony
is required to establish that the events leading to Ps injury would not likely
happen without carelessness

k.
l.

m.

n.

o.

p.

i. Would bar RIL if expert was arguing specific precaution that could
have been taken
RIL invoked in other caes
i. Modern trend to let airplane passengers killed in airline crashed with
no evidence as a means of establishing pilot fault
Exclusivity prong
i. Modern trend dont read it as absolute control, but enough to show
that D likely to be the only one to have undertaken or omitted relevant
acts
RIL against Multiple D
i. Ybarra v Spangard (p. 224, 1945): P underwent appendectomy woke
up with shoulder paralysis. Theory was positioning during surgery
a. RIL allowed to establish carelessness on the part of several Ds
b. Duty applies to med mal with team communicating
c. Team was treated as single actor
i. Shifts burden to Dr. and forces them to come forward as to
who was acting negligently
d. LIMIT: rarely (if ever) applied outside of medical malpractice
(need formal relationship between the Ds)
ii. James v. Wormuth: res ipsa did not apply:
a. both because other medical personnel were involved in the process
of inserting the wire and transporting the patient (thus defeating the
attribution of exclusive control to the surgeon)
b. and because the alleged malpractice involved an intentional
decision to leave an object in a surgical patient rather than
inadvertance.
Plaintiff Participation
i. Arguably just an extension of control requirement
ii. If plaintiff played a role the likelihood of Ds role in bringing about
the injury sifnificantly decreases, baring jury from RIL presumption
iii. Some courts have explicitly dropped this requirement
Spoilation of Evidence:
i. If, after an accident, the defendant intentionally (or, in some states,
negligently) destroys evidence tending to establish its carelessness as
the cause of plaintiffs injury, the plaintiff may have a separate cause
of action against the defendant for spoliation of evidence.
Res Ipsa Scenarios

C. ACTUAL CAUSE
1. BASICS
i. Jury question; preponderance of the evidence (>50% likely that Ds
breach needed to happen for Ps injury to have occurred)
ii. Must be a tortious act, not something non-negligent
iii. Threshold tends to be very low for purposes to get to the jury
iv. Judge has gate keeping role (rigorous threshold)
a. Old Common law approach
i. Used as proxy
v. Ways of aiding fact finder
a. Substantial factor
i. Limited by R2 and R3
vi. Courts have found ways to put thumb on scale for P
a. Does not mean P automatically recovers but allows to move
forward
i. Statistical evidence and expert witnesses
ii. Relaxed causation
iii. Multiple unidentifiable tortfeasors
b. Policy Rationale
i. Corrective justice: ONLY wrongdoers should have to pay for harms
ii. NOT about deterrence (if this were point, why not have all speeders
pay for accidents, even if didnt cause)
iii. NOT really about compensating accident victims (many cant clearly
prove who did wrong/how it happened, so never compensated)
c. P must prove that Ds breach of duty probably functioned as an actual cause of
the Ps injury
d. But-for (sine qua non (without which not) or counterfactual test
i. Cause-in-fact or factual cause
ii. Asks: Did Ds carelessness play a role in bringing about Ps injury?
a. If no causation is established, if yes Ps claim fails test an no
causation

e.

f.

g.

h.

iii. A Cause not the cause


a. Does not necessary entail the conclusion that A is the only person
whose carelessness played a role
Substantial Factor
i. Many courts use this term in the descriptions of the test
ii. R2: 431 "for careless conduct to be a "legal" cause of harm it must
be a "substantial factor" in bringing about the harm")
a. Does not identify a separate test but used to clarify the but-for test
iii. R3: (Drops substantial factor because confusion)
a. Tortious conduct must be a factual cause of harm for liability to be
imposed. Conduct is a factual cause of harm when the harm would
not have occurred absent the conduct.
i. A separate provisions in the Physical and Emotional harm
section of the Third Restatement 27 maintains that the butfor test should not be used as the test for actual causation
in the rare scenario in which a D's carelessness operates
as one of two or more sufficient causes of the P's injury
Proving AC under the Preponderance Standard
i. Ordinarily a jury question
ii. P must prove Ds breach probably needed to happen for P to be
injured
a. Absent reliable eyewitness or video
i. Circumstantial evidence that is sufficient to permit a
reasonable factfinder to infer a probable causal connection
between the defendants carelessness and the Ps injury
ii. Courts will sometimes permit findings of causation
notwithstanding an apparent lack of evidence sufficient to
meet the preponderance standard
But-for Traps
i. Post hoc ergo Proctor Hoc
ii. Correlation = Causation
iii. Live in a world with many causes
iv. Unknown causes/Unidentifiable causes
Mucker v. Buchl (P 234, 1967):P fell down flight of stairs and died where D had
control of switch and staircase was dimly lit
a. More probably that the darkened state of the stairway was the
precipitating factor for the accident
Rule: An even which could be an adequate cause of an accident can be
deemed the cause if the probabilities are great enough
i. Court guided by three theories and plying monitoring role

i.

j.

k.

l.

m.

Statute: Violation of statute about lighting itself can establish


causal connection
ii. RIL: Exclusive control, more likely than not
iii. But for Cause: not having lighting in common are-> substantially
increases risk of harm
Butts v. Weisz (P 238, 2010): Ps wife fell in Ds house in PA going to the
bathroom where the D had turned the light off near a stairwell
i. Rule: Increased risk of harm due to a Ds negligence, standing alone,
does not permit an inference that an injury, more probably than not,
was caused by the negligence
a. Uses Fedorczyk standard where a passenger on a cruise ship
slipped and fell in bathtub because not big enough non-slip strips
ii. Careful reading: AC cases can go to jury even with claims of
comparative negligence (doesnt bar recovery anymore)
iii. Some jurisdictions would use RIL here, not used here
iv. PA Standard:
a. Extremely low threshold for jury determining cause
b. Substantial factor
Judge v. Jury
i. AC generally question for jury
ii. D has plenty of opportunities to have judge take actual cause away
from jury
Daubert Screening
i. Gate keeping role of judge
ii. Expert is testifying in area where expert
a. Filters out junk science and stats
What must be Proven and What need not be Proven
i. Must:
a. P must show their account is the more probably account
b. It is likelier than not that she would not have been injured had the
defendant been careful
ii. Must Not:
a. Need not show Ds proposed account could not have happened
b. Need not show she could not have been injured if D had acted with
care
Kallenberg Approach: the court upheld a jury verdict against the defendant
hospital even though it acknowledged that the plaintiffs own expert had
testified that proper treatment would have given the plaintiff a 20, say 30,
maybe 40% chance of survival (rather than a greater than 50 percent chance of
survival).

i. The Kallenberg approach does not relieve a plaintiff of the burden of


persuading the factfinder that the defendants conduct was probably
necessary for the plaintiffs injury to occur.
ii. Gives juries substantial leeway to infer a probable but-for connection
between breach and injury from a combination of statistical and
circumstantial evidence.
iii. Lowers the P's burden of production on actual causation yet
maintains the usual burden of persuasion
n. Substantial Factor
i. CA: but-for must be established before substantial factor jury
instruction
a. Refines and clarifies but-for test
b. Most courts approach
ii. PA: lower threshold, more P friendly
a. Hamil: As long as action functioned as a substantial factor (by
somewhat increasing the risk) P can still prevail
b. explicitly lower the burden of persuasion, allowing the plaintiff to
recover in a case in which the defendants carelessness increased
the risk of the plaintiff being injured, yet the injury probably still
would have occurred even if the defendant had been careful.
o. Three approaches recap
i. Kallenberg: Lowers burden of production, but maintains persuasion,
Ds carelessness probably was necessary for the P to have been
injured
ii. Hamil: recovery if Ds carelessness increased the risk of P being
injured, yet the injury probably would have occurred even if the D had
been careful
iii. Loss of Chance Doctrine: (Med Mal) permits a claimant with a 50
percent or less chance of survival or better outcome to nevertheless
recover damages for any negligence of the physician that reduced this
chance of survival or better outcome.
a. Medical malpractice only, Matsuyama
b. Harm is P loses a chance of survival because of Ds negligence
c. I.e. P proved that because of Ds negligence, descendent lost a
37.5% chance to survive
d. Threshold: Reasonable degree of medical certainty
e. D must be more probably than not the but for cause of this loss of
descendants chances
f. Jury must find that its more than likely than not that expert is right
about probabilistic claim

g. Many states dont recognize this. >50% lost chance = 100%


recovery
h. <50% lost chance recover but damages are adjusted
[proportionate causation based off life expectancy]
p. Causation and Breaches of Affirmative Duties
i. Often courts are lenient with causation in breaches of affirmative duty
cases
ii. U.S. Supreme Court has held that when the defendant breaches an
affirmative duty to disclose material information, a court may
presume that the persons to whom the duty was owed would have
changed their conduct had the information been provided.
2. MULTIPLE CAUSES
a. Multiple Necessary Causes: McDonald v. Robinson (p 260, 1929): Two cars
hit each other and slid into a pedestrian, causing severe injuries
i. Rule: If the acts of two or more persons concur in contributing to and
causing an accident, and but for this happening the accident wouldnt
have happened, the injured person may sue the actors jointly or
severally, and recover against one or all
a. Single Indivisible injury
ii. Reaffirms:
a. Inquiry for actual cause at gate keeping level and burden of inquiry
A Cause not The Cause
b. Doctrines of joint and separate liability
i. Not a causation doc but a damages doctrine
b. Relation to Prox Cause
i. Can be actual cause but this does not necessarily say each should be
held responsible or liable
c. Damages
i. P can collect 100% from one D if other is insolvent, but no more than
100% overall
d. Affirmative duty
i. Breach here can count as one of two or more but-for causes
e. Actual Cause in Toxic Torts
- But for test & preponderance standard, combined with Daubert make it
difficult for TT ps to prove AC
- Courts have given juries less leeway to infer AC in TT cases
Three Dimensions in TT cases
i. General Causation

a. Can prove that substance S is probably capable of causing


disease D in humans?
ii. Exposure at a sufficient Dose
a. Can prove that she probably was exposed to S at a level
sufficient to cause her disease?
iii. Specific Causation
a. Can prove that her exposure to S probably was necy for her
disease?
Have all 3 generally enough to survive SJ and get to jury
3. SUFFICIENT CAUSES
a. R2 Substantial Factor and Multiple Sufficient Causes
i. Use substantial factor language was to explain result of multiple
sufficient causes
ii. An actors carelessness must be a substantial factor in bringing about
an injury in order to be deemed a legal cause of that injury
iii. Carelessness substantial factor only if
a. A non-trivial necessary condition
b. One of the two force is each sufficient to bring about harm to
another
iv. Does not displace but-for test
v. Multiple Sufficient Causes Anderson (P. 284, 1920): Illustrates
second bullet of restatement
a. Two fires destroyed Ps property
b. Each fire was itself of sufficient magnitude
c. Not all cases involving multiple sufficient (duplicative causation)
causes trigger substantial factor test
i. Only when there is temporal concurrence between the force
carelessly created by the D and the force of unknown origin
that the substantial factor test was to be used (Concurrent
Causation)
d. R2 says substantial factor is appropriate with this
e. R3 rejects SF test but acknowledges one exception in this case
where cant meet but-for but still find causation
f. ONLY EXCEPTION WITH SF in R3
g. Today most courts begin with but for
i. Have relaxed statistical probability significantly to find
cause
b. Do we need NESS Test?
4. ALTERNATIVE LIABILITY/CAUSATION

a. Summers v. Tice (p 288, 1948): Three men hunting and P gets shot by one of the
Ds who negligently fired at the same time
i. Rule: When P cannot determine which of multiple negligent Ds
caused his injury, all Ds are liable, and it is up to each D to absolve
himself (Alternative Causation)
ii. Rationale:
1. Shifts burden to D
2. Wronged party should not be deprived of redress
3. Creates incentives to come forward and reveal evidence
4. Unknowable factor for P
iii. Applies in situation in which one of two or more negligent actors acts
independently of the other to cause injury to the plaintiff
b. Three other scenarios
i. Multiple Actual Causes: Independent careless conduct of two actors
each functions as an actual cause of Ps injury
a. Two careless acts, Summers was only one
ii. Conspiracy: Existence of plan working together to injure P
iii. Concert of Action: Treats tortfeasors as acting together, but dont
need plan.
a. i.e. Drag racing, one car hits someone, both are liable
5. MARKET SHARE LIABILITY
a. Basics
ONLY imposed in DES context
- Fordham Law student took concept from Calebresi
- In some ways collapses distinction between causation and apportionment
Key Factors:
i. (1) ALL identified Ds are potential tortfeasors
ii. (2) Product must be fungible
a. Usually involve latent risks that appear later
b. Each D had the same likelihood of causing the same amount of
harm through each pill (generic drug, same dosages)
c. Market share becomes a good proxy for overall harm by each
manufacturer.
iii. (3) P must have sued a substantial share of the market
iv. (4) P must not be able to identify which D is responsible through no
fault of Ps own
b. Policy Arguments FOR
i. Not joint and several liability
ii. Each manufacturer is liable for damages equal to their share of the
market

iii. Allows for compensation and deterrence (economic approach)


a. BUT turning Tort law into social welfare mechanism (taxes better)
iv. Burden-shifting (burden shifted to D) b/c too burdensome for Ps
(would get away w/out paying otherwise)
v. Efficiently distributes losses that arise from negligently manufactured
products (manufacturers can get insurance, increase price of goods)
a. BUT also leads to deep-pocket searching
vi. Manufacturers are the cheapest cost avoiders of the injury (efficient
deterrence)
vii.
Alls Ds responsible for injuring some people (just not
necessarily these people)
c. Policy Arguments AGAINST:
i. Impractical/costly to administer
ii. Allows victims to recover from Ds who may not have harmed them
iii. Not uniformly the case that toxic substances cause signature diseases
(why not used more)
iv. Task better suited to legislature or executive branches (social
insurance scheme), BUT class actions do same thing and arent
viewed as negatively
v. Tort law is now about reducing tort liability (v. when Sindell decided)
d. Sindell v. Abbot Labs (P. 293, 1980): women are suing DES manufactures
because their mothers took the drug during pregnancy and now they have many
health issues; limited time frame when drug was produced, class action, Ds
represent 99% of drug manufacturers, drug was identical between Ds and
women cannot tell which D they got the drug from (impossible to tell)
i. Rule= Market Share Liability: if, through no fault of her own,
cant identify the mfr who injured her & sues mfrs who, together,
constituted a substantial share of the relevant market
ii. NOT:
a. Alternative liability b/c not all possible wrongdoers were joined
(the 1% that was not represented could have manufactured the drug
that Ps mom took) AND no simultaneous activity (like Summers)
AND no EQUAL chance that each D did it (like 50% in Summers)
b. Concert of action liability (ex: drag race - everyone understands
they negligently acted together);
c. Industry-wide liability (Halls case: blasting cap manufacturers held
liable for the standards they set up; oligopoly, with safety standards
board for all)

e.

f.

g.

h.

iii. In some states, Ds can avoid liability by showing they could NOT
have caused the injury to the P (due to not selling at the time, in the
state, etc)
Market Share Liability Outside CA
i. NY:
a. Hymowitz
i. "savings" statute revived claims that would otherwise have
been time-barred
ii. Creates an irrebuttable presumption on causation under
which each D is held liable to each DES P in proportion to
its market share even if a given D could show that its DES
could not have caused a particular Ps injury (unlike Sindell)
iii. Also market share is determined by national market not
geographic market
iv. Dont allow mfr to prove that their pill wasnt the ingested
one
v. MARKET SHARE IS SEVERAL LIABILTY
i. Cant recover 100% from one D
ii. Can be joint and several
ii. FL:
a. Conley: market share should be calculated at a geographically
narrow level
iii. WI:
a. Collins: D pharma companies are joint tortfeasors, market share is
deemed relevant to the jury's apportionment of responsibility
among them (Risk Contribution)
Intergenerational Market Share
i. Enright (Ny 1991): Court says manufacturers could not be charged
with a duty to be vigilant of the effects of its product on thirdgeneration victims
Beyond DES
i. MTBE: gas polluted groundwater
a. D.Ct determined that the substantive law of several of the other
jurisdictions would permit the claims to go forward on market
share theory
ii. Thomas: Child with mental disabilities resulting from lead paint
ingestion
a. Using Collins denied SJ motion by D
Where do we set bar for causation?
i. Corrective Justice (higher bar)

ii. Deterrence or loss spreading (lower bar)


a. Doctrines like market share are to achieve a purpose
b. DES and Hymowitz
iii. How we determine breach makes a big difference
a. If RIL no need for but for test or causation determination
i. Apportionment
i. Causation different than apportionment
ii. Market share collapses in a way
iii. Sindell leaves open apportionment question
iv. Would liability imposed be joint and several?
a. 8 years later CA court says NO
j. DES and Cigarettes?
i. Not fungible
ii. Risk not latent
iii. Marketed differently
iv. Choice of Consumer
D. PROXIMATE CAUSE (LEGAL CAUSE)
1. Basics
a. s breach was an actual cause of s injury, but did it cause the injury in the
right way?
i. Did breach cause the injury in a natural or expected rather than
serendipitous manner
b. Align the elements- Show duty and breach connected
c. Need both prox cause and actual cause to recover
d. P not pleading proximate cause, usually D responding to show not prox cause
e. Often called legal cause because most issues with proximated cause are
decided by the judge
f. Connection / overlap between proximate cause and duty
i. Hard to distinguish proximate cause cases from duty cases
ii. One difference is who decides
a. Duty judges
b. Proximate cause jury
iii. Distinguish by thinking about what they focus on
a. Duty asks to decide whether this D should be liable to this P or
class of Ps (who question)
b. Proximate cause asks the what question for what injuries
should D be held liable?
2. Tests

(1) Natural and ordinary consequences (ex: fire - one leap rule)
a. No longer used
b. Natural and ordinary for fire to travel from its source to one other structure, it is
neither natural nor ordinary to spread further
c. Remote consequence
(2) Directness tests- Ps injuries had to flow directly from Ds negligence
d. No longer used
e. Polemis (1921) -- workers dropped plank into ship causing explosion (liability
found, though not foreseeable, because direct result)
i. Fire not foreseeable but direct continuous effect flowing from
negligence
(3) Foreseeability test (dominant modern test)- Ps injures must be seen as
reasonably foreseeable ex ante at the time D acted; D should have foreseen those
injuries
f. Wagon Mound I- owner of neighboring dock sues a ship that pours oil into
harbor; it was seen as unforeseeable and no liability found
i. Rejects Polemis
g. Wagon Mound II-- another ship owner sues the ship that poured the oil; the
ship-owner had foreseen the fire (though assured by dock owner it wouldnt
catch fire); it was found foreseeable and liability was found
i. Different Ps, one was better positioned to develop foreseeability
evidence
(4) Risk-rule test (3rd restatement test) - was the injury to P within the scope of
risk(s) that made D's action negligence in the first place
h. A lot like neg per se
3. Union Pump Co. Allbritton (P 311, 1995): Woman slips on a pipe after putting out an
emergency fire and injures herself
a. Unusual in Texas, Proximate cause is a doctrinal issue
b. Proximate cause is Actual Cause + Foreseeability
c. Forces here had come to rests
d. Rationale: court seems to feel that Allbritton was contributorily negligent or
assumed the risk by walking over a pipe rack that was known to be dangerous.
i. Using proximate cause to draw a line where the judge thinks P is at
fault
ii. Dissent argues correctly that the issue of contributory negligence
should be considered separately
e. Issue of whether the emergency is over
i. Majority thinks the fire is ended, and liability only extends to injury in
the context of the emergency

ii. Dissent thinks the fire / emergency hasnt ended no real


disagreement on verbal formulation of the law
f. Modern comparative liability makes a lot of Union Pump concepts irrelevant
because about how we apportion responsibility
4. Ventricelli: rental car with defective trunk
a. Collision was not foreseeable because parked in a relatively safe place
Was P a foreseeable victim?
5. Palsgraf v. Long Island Railroad (P 330, 1920): Passenger trying to get on train is
pushed up by train employee and drops fireworks that explode and cause a scale to hit
a woman in the train station
a. Cardozo:
i. Jury could not find that D was careless with reference to P
ii. A negligence P cant piggyback on s carelessness toward a differently
situated person
b. Cardozo thinks this is not prox cause but a duty case, and no breach
c. Corrective justice individuals who are wrongdoers owe a duty of repair to the
persons they wronged and only to the persons they wronged
d. Andrews (dissent): there is duty, and its a proximate cause question (AND in
the end there is liability here)
i. Duty owed due to common carrier and public space
ii. Everyone owes duty to the world at large
iii. Public Policy and Expediency
iv. Andrews dissent finds PC present because direct and continuous
sequence of events.
v. However, note that this is not the test, under Wagonmound or the
Restatement.
vi. In negligence cases, you must analyze foreseeability of the manner
in which the accident occurred ask is the harm within the scope of
the risk
vii.
viii.
Rossi agrees with Andrews analysis BUT finds no proximate
cause and no breach
ix. Foreseeability test of proximate cause
x. What we . . . mean by the word proximate is that, because of
convenience, of public policy, of a rough sense of justice, the law
arbitrarily declines to trace a series of events beyond a certain point.
This is not logic. It is practical politics.
xi. jury ordinarily has leeway to apply common sense and should have
been given that leeway in Palsgraf

6. Posner says most important lesson is that Palsgraf says that efforts to employ legal
concepts such as duty to resolve difficult cases are hopeless
7. Wagner: P and cousin were standing in a tram and cousin was thrown out and P
walked back along the track and fell
a. Cardozo: says doctrine of danger-invites-rescue applies to any rescue effort
this is carried out both reasonably and contemporaneously with the carelessly
created peril
i. Leaves it to jury where P was barred from suing under contributory
negligence
8. What is the source of the relationality requirement?
a. In tort, [a]ffront to personality is still the keynote of the wrong.
b. The victim does not sue derivatively . . . to vindicate an interest invaded in the
person of another. Thus to view his cause of action is to ignore the fundamental
difference between tort and crime. He sues for breach of a duty owing to
himself.
c. If duty is about defining relationships between the parties for purposes of fixing
responsibility, then proximate cause should not be a way of second guessing a
finding that duty existed.
d. Note that this also has implications for the analysis of breach. If the duty is
relational, then breach is not just a general wrong, but a wrong toward the
plaintiff.
i. I.e., the conductor could have acted wrongfully w/re: a passenger, by
aggressively pushing or pulling someone onto the train, but this is not
a wrong vis--vis Mrs. Palsgraf.
9. Androzo I:
a. Cardozo reaches right result for the right reason but misstates the doctrinal hook
for applying those reasons
b. Andrews properly framed the question but then reached the wrong result by
misunderstanding proximate cause to torn on inquiry concerning expedience
(instead of the narrower scope of the risk question)
c. This approach has been adopted by the R3
10.Androzo II
a. California and NY have melded Cardozo and Andrews another way
i. Andrews was right to say this question cannot be resolved by
employing a subtle understanding of how the elements of negligence
align with one another
a. Should take an all things considered approach as to whether
expedience or policy suggests that the defendant ought to be let
off the hook notwithstanding that is carelessness caused Ps injury

b. Agree with Cardozo that expediency should be asked under the


heading of duty and not proximate cause
c. Andrews ask roughly the same question as Rowland v Christian
i. Yet done under duty not prox cause
d. Becomes question of law for the courts
11.Scope of the Risk
a. Leading alternative to foreseeability and prox cause
b. A careless actor is subject to liability for harm actually caused by his careless
conduct if the harm is a result within the scope of the risks by reason of which
the actor is found to be negligent (Keeton)
c. R3: adopts scope of the risk by framing proximate cause inquiry in terms of
whether the injury suffered by the victim is one of the harms whose risks
rendered the actors conduct careless
12. Superseding Cause
a. The subsequent act of a second tortfeasor can sometimes relieve an earlier
tortfeasor of his responsibility, even if he is a but-for cause of the victim's injury
i. Two wrongdoers are acting independently of one another
a. No concert of action or common plan
ii. Their wrongs are committed in a sequence, such that there is an
initial wrongful act that is more remote in time and space from
the Ps injury and a subsequent wrongful act that involves a
relatively direct or immediate infliction of injury on the P
b. Standard: to be a superseding cause (and relieve an initial tortfeasor of
liability), the intervening act must be so highly extraordinary that
antecedent negligence should be ruled out as a matter of law as a substantial
factor in causing the accident.
c. Invokes some notion of reasonable foreseeability
d. When the risk of injury to the P is so unforeseeable that it is not a factor to be
taken into account at all in deciding whether the Ds actions were negligent, the
subsequent act of the 3P is described as a superseding cause
i. As a matter of law the negligence of the D is not a PC cause of the Ps
injury
ii. If harm or manner foreseeable (given risk created by D, then no SC)
iii. A lot depends on how you frame that first wrong (Foreseeability
regarding the same type of harm)
a. YES PC (no break in causal chain) dropping girl off in sketchy
part of town, foreseeable she would get raped not SC
b. NO PC (break in causal chain) having not trimmed bushes, not
foreseeable someone would get raped behind them (just
intervening, not SC)

c. Fire by arsonist, P liable (no break in causal chain)


i. Negligence goes to failure to provide sufficient lighting
directly relates to risk associated with fire so actually
doesnt matter
e. Some courts require the second cause to be intentional but not always
i. R2 says that second harm has to be intentional and not within the
scope of the risk (why Manheimer/bushes wasnt PC, but Hobos
wasnt)
ii. R3 goes even further and says intervening actors who commit
intentional torts are not superseding causes if the harm that occurs is
within the scope of the risk
f. Why let P1 off the hook?
i. Fairness
ii. Cheapest cost avoider
g. Acts of God usually count as superseding causes
h. Still Used? Many argue that with apportionment in comparative fault, this is
outdated
13.Pollard and Clark
a. Pollard v. Ok. City RR kid found and collected dynamite along the side of Ds
railroad. He and another kid put it together and lit it, and the other kid was
badly burned
b. Clark v. EI Du Pont P found a piece of solid glycerin and took it away to
prevent injury. He hid it in a cemetery, where kids dug it up later.
c. Underlying doctrinal issue is the same in both cases: whether the harm was
foreseeable
d. Why can P recover in Clark and not in Pollard?
i. Nature of the interving conduct exacerbates in Pollard and lessens
in Clark
ii. Several intervening actors in Pollard parents failure to supervise
their child
iii. Ps knowledge in Pollard, P knew it was dynamite and would blow
up
iv. Degree of negligence in Pollard
a. Workers knew the kids were gathering the dynamite court
doesnt focus on this, but its a key exacerbating factor
b. Maybe this case came out wrong
e. R3 relies on apportionment and rejects superseding cause
f. Port Authority of NY & NJ v. Arcadian Corp (P 361, 1999)
i. building owner v. fertilizer manufacturer (mfr)
ii. is not suing terrorists

g.
h.
i.

j.

k.
l.
m.

iii. Misfeasance: mfr carelessly damaged my building


i. carelessness = selling product that can easily be made into
bomb
ii. does terrorist use of product count as a superseding cause?
i. yes, this use was unforeseeable (?) ergo no proximate
cause
ii. i.e., the terrorists introduced a new power of doing
mischief
iv. Nonfeasance: mfr carelessly failed to protect my building
a. what happens to the superseding cause issue?
b. from frying pan to fire?
v. a duty to protect owed by to would solve the superseding
cause issue, but there needs to be such a duty and there isnt
a. contrast a tenants suit against landlord for lobby attack
b. Breach of affirmative duty gets rid of SC issue
Per Se Rule
i. Modern trend has been to abandon this rule that required SC to be a
criminal act and do a foreseeability inquiry
Dram Shop acts
i. Commercial businesses liable when serving minor or drunk people
who may drive and hurt someone
Gun Litigation
i. No duty with manufacturaer with victims of shooting
a. Cardozo rationale
i. Victim not foreseeable
ii. Public policy concerns
Social Host liability
i. Most courts say cant sue host who provided guest alcohol and guest
kills someone in drunk driving accident
a. Is victim foreseeable, within Scope of the risk
i. No, except with minors
Liability of premises for criminal attacks
i. Widely followed that landlords are responsible to take reasonable
security measures to protect tenants in common areas
In h,I,j,k duty doing most of the proximate cause work, robust analysis on
relationality
Aligning the elements Recap
i. Relationality (Duty +Breach): must prove not merely that was
careless, but that was careless as to

a. Andrews Palsgraf dissent: there should be no relationality reqt


but instead commonsense limits on liability
ii. Proximate Cause (actual cause + injury): must prove s breach
caused s injury in a non-fortuitous manner (fortuity determined by
4Cblty or S.o.R. test)
a. Andrews Palsgraf dissent: prox. cause gives jurors an occasion to
set commonsense liability limits
E. DAMAGES
1. Goal is making P whole/ restoring the status quo ante
a. However, make whole not really the standard juries told what they deem fair
and reasonable in light of theirfindings
b. Eggshell skull rule D is liable for the full extent of damages, even if the
extent is unforeseeable
i. Vosberg v. Putney
ii. Smith v. Leech Brain molten metal burn on lip became cancerous
and killed P
iii. Jury instructions vary on whether this is permissive or mandatory
iv. Mustapha (eggshell psyche):
a. Defense that reaction was unforeseeable is not allowed
v. R3: Actors conduct must be such that would cause a reasonable
person to suffer serious emotional harmhowever once that
threshold is met a person may recover for all harm subjectively
suffered, even if that suffering is greater than an ordinary persons
because of a predisposition of special vulnerability
2. Three Types
a. Nominal: a token for a tort without loss
b. Compensatory: Compensation for what p has suffered at hands of D
c. Punitive: $ owed by D to P as punishment for egregious mistreatment, further
deter
3. COMPENSATORY
a. Economic Damages
i. Loss of income
ii. Medical Expenses
b. Non-Economic Damages pain and suffering
i. Not available under workers comp
c. Advantages of compensatory damages: juries can respond to the particular case
before them
d. Disadvantages:

e.

f.

g.

h.

i.

i. Jury influenced by racial biases and socioeconomic biases


ii. Jury awards can be unpredictable/arbitrary; thus not deterrence.
Smith v. Leech Brain & Co (513, 1962): Molten metal burns Smiths lip and he
dies of cancer related to the burn
i. Clear statement of eggshell skull rule
ii. ct: can recover damages related to death
a. tortfeasor takes his victim as he finds him
b. proximate cause limits liability not damages
iii. Can prox. cause & damages be distinguished?
a. recall Wagon Mound unfair to hold liable for un4cble injury
(fire) just because difft injury (mucking) was 4cble
Kenton v. Hyatt Hotels Corp (P 521, 1985):
i. Holding: the evidence is let in, and plaintiff awarded $800K for pain
and suffering over and above her economic losses
ii. Court says this award isnt excessive recognizes that the results
seem arbitrary, but says anything is ultimately going to be arbitrary
iii. TEST: What fairly and reasonably compensates plaintiff for the
injuries sustained
iv. Economic damages (lost earnings ($ 2.2 mill, $ medical costs $ 1
mill)
v. Noneconomic damages (pain and suffering $.8m)
Bifurcation
i. State and federal law typically allow the judge broad discretion to
bifurcate the trial of a tort action based on her assessment of whether
doing so will promote the efficient and fair resolution of the
dispute before her
a. Prevent bias
Judge review of damages
i. Longstanding black letter doctrine holds that a trial judge should
not second-guess a jurys compensatory damage award unless, after
viewing the evidence in the light most favorable to the plaintiff, the
verdict shocks the conscience or is so out of line with the evidence
presented that it was likely the product of passion, prejudice or other
improper motive.
Remittur and Additur
i. Remittur: can accept lower amount in lieu of new trial
ii. Additur: Some jurisdictions empower the trial court to offer the
defendant, in lieu of a new trial, the option of paying a specified larger
award (again, only upon a finding by the judge that the jurys verdict
is shockingin these instances, shockingly low).

j. Appellate review
i. Standard for damages is did T.CT Abuse its discretion?
k. Collateral Source Rule: D cannot introduce evidence that victim has received,
or stands to receive compensation for her injuries from some other source
i. Rationale:
a. Tortfeasor should not benefit just because P had insurance
b. Others say to deduct for insurance proceeds would deny P benefit
she has secured from insurance
c. Other argues helps counteract the American rule denying
successful P attny fees as part of damages
ii. Modern trend: at least half the states have modified or abolished this
rule
l. Pain and Suffering Damages and Tort Reform
i. As courts, commentators, and interest groups, and legislatures have
become increasingly concerned about the size of jury verdicts,
legislatures have responded with various measures, including some
that directly address compensatory damages
a. VA limits total compensatory damages
b. Other states have enacted legislation placing a cap only on
noneconomic losses (CA Civ Code)
c. Others eschew caps in favor of more robust judicial review (NY
law)
ii. who bears the burden of this sort of cap?
a. the most traumatized s? s w/ minimal econ. losses (e.g.
retirees)?
4. PUNITIVE DAMAGES (exemplary/vindictive)
a. Basics: Willfulness/Wantonness/consicious disregard
i. Judge can have role in determining if damages are excessive
a. Common law standard: reasonable juror
b. Shocks conscious: appellate court very deferential to trial court
ii. Jury-Just because can award PD does not entitle P to them
automatically
a. Only if tort and willful, wanton, reckless actions
b. Lots of discretion as opposed to CD
c. Jurisdictions split on BURDEN OF PROOF
i. Many say P must present clear and convincing evidence
showing quite likely D acted with the requisite disposition
iii. Intentional Torts
a. Often, but not always available

i. playfully kisses , a stranger, whom rsbly mistakes for


his partner: even if a battery, no PDs
b. Probably most often awarded for business torts (e.g., fraud)
rather than personal injury claims
iv. Negligence
a. who proves only carelessness is ineligible for PDs (not malicious
or wanton)
b. But a negligence who can prove carelessness that rises to the
level of wantonness is eligible for PDs
Factors for determining amount of
punitive award
Reprehensibility of Ds conduct
Comparison of harm D might have
caused to harm P actually suffered
Jacques
Wealth of D

Purposes of awarding punitive damages


b. Deterrence
i. Compensatory damages might not always provide sufficient
deterrence, i.e. if the kind of wrongdoing is hard to detect, the amount
awarded wouldnt be enough to encourage litigation
ii. In order to induce more litigation, need to provide people with the
possibility of punitive damages
a. But punitive damages arent awarded very often for them to play
a substantial deterrence function, would probably have to be
awarded more often and awarded in a more predictable/consistent
way
b. Punitive damages are awarded rarely, and usually to plaintiffs
asserting claims for intentional wrongs (i.e. battery, assault, fraud)
c. Retribution
i. Damages can be seen as sending a message to def
a. But this rationale starts to resemble deterrence
ii. When def is a corporation (rather than an individual), less likely that
such a def will be susceptible to retribution (though may say
corporations are concerned about their corporate personality)
d. Compensation
i. To the extent that it is difficult to value dignitary harms through
compensatory damages, punitive damages can supplement may
achieve a fuller measure of compensation

e. Probability of detection
i. Argument that you need to make def pay more when its harder to
detect the wrongdoing, because def wont always be caught
a. So punish them when they are caught
b. Also works in terms of an incentive for victims to sue (see under
deterrence)
f. Substitute for private parties taking the law into their own hands
g. Release mechanism punitive damages are a means of extracting a form of
vengeance
5. When to Award
a. Instances when to award punitive damages:
i. 1. When compensatory damages are too low to deter wrongdoing b/c
it is easy for the wrongdoer to avoid detection (Mathias bedbug case)
- need a certain quantum of people suing to have effect OR punitive
damages.
ii. 2. When D is likely to turn a profit from wrongdoing. (Mathias again)
iii. 3. When violation is non-economic in nature and only leading to
nominal compensatory damages, so not enough deterrence (spitting in
face)
b. TEST FOR PUNITIVE DAMAGES (National Byproducts)
i. NEED AT LEAST RECKLESSNESS.
a. Gross negligence? Dunno.
b. Regular negligence does not hold up, naturally.
ii. Evidence indicates that the D acted WANTONLY in causing the injury
or with such CONSCIOUS INDIFFERENCE to the consequences that
MALICE may be inferred.
a. Malice = very likely to get punitive damages.
i. Distinct from intent, malice is a bad motive, acting out of
spite/ill will
b. Dissent emphasizes: Malice no longer needed: enough if D acted
recklessly or wantonly or with conscious indifference to the safety
of others.
iii. ***For reckless, willful, or wanton conduct***:
a. 1) Conduct has to create a high degree of risk of harm (P) OR
risk of very serious harm (L) AND
b. 2) D, when acting, must be conscious of the risk and must proceed
without concern for the safety of others. (Mental element beyond
regular negligence)
iv. Gross negligence is a situation with high P or L and no conscious
mental element.

v. From Mathias: Willful


c. MATHIAS Factors to take into account when deciding whether to allow award
punitive damages:
i. 1. Reprehensibility of Ds conduct
ii. 2. Ratio between harm caused by conduct (including noneconomic
harm) and the economic losses suffered/granted
iii. 3. Wealth of D
iv. 4. Profitability of tortious conduct
v. 5. Ps financial cost in bringing suit
vi. 6. Probability of detection of tort
6. National By-Products v. Searcy (p 537, 1987): Truck stopped under a bridge, National
employee was speeding in a truck and hit another car, which hit Searcy
a. Award justified only where the evidence indicated that D acted wantonly in
causing the injury, or with such a conscious indifference to the consequences
that malice can be inferred
i. Wanton disregard = plaintiff has to show that defs conduct posed a
grave danger of harm to others, and that def had reason to know facts
that would lead a reasonable person to realize the dangers of his
conduct
a. Slightly lower threshold for getting punitive damages def doesnt
actually need to be aware
ii. Conscious indifference = plaintiff has to show that def was aware
that his conduct created an unreasonable risk of physical injury to
others, and the risks were substantially greater than necessary to make
it careless
a. More careless than will constitute a breach of the standard of care
requires a conscious choice of a course of action that recognizes
it involves a more substantial risk than mere negligence
iii. Dont need to prove actual malice
7. Mathias v. Accor Economy Lodging (p 541, 2003)POSNER: Ps bitten by bedbugs
while staying at a motel and motel knew about the problem but continued to rent
rooms for months
a. awarding punitive damages in this instance isnt excessive
i. Juries have discretion to award punitive damages in subset of
negligence cases gross and criminal negligence
b. Reckless indifference distinguished from mere carelessness
i. Can be wanton disregard or conscious indifference (Searcy
distinction)
c. What do PDs accomplish

i. traditional view: allow victims of egregious wrongs to make an


example out of (exemplary damages)
ii. L&E view: prevent under-deterrence of acts that are:
a. not likely to be prosecuted criminally
b. cause minor injuries (no one has incentive to sue for
compensatory damages)
c. difficult to detect (e.g., fraud), and/or
d. undertaken by pertinacious (determined) s
iii. Economic view is controversial (casebook doesnt buy it)
a. Opening up compensation to Ps recovering over and over for same
conduct
b. If deterrence is goal then why willful and wanton standard?
8. Recklessness R3:The actor must not only impose an easily avoidable significant risk
of injury on others, but must be consciously indifferent to the risk or wanton in
disregarding it
9. Deliberate Indifference R2: the actor to be reckless must recognize that his conduct
involves a risk substantially greater in amount than that which is necessary to make
his conduct negligent
10.Punitive damages and Tort Reform
a. State-Law Reforms
i. bifurcation/trifurcation: separate trial on PDs
ii. robust judicial review of jury PD awards
iii. safe harbors for regulatory compliance
a. if manufacturer complied with all regulations some statute bar PDs
iv. splitting statutes: x% of PD award goes to state
b. Constitutional Limits and Ds 14th Amendment Due Process Rights
i. Haslip (US 1991)
a. fraud claim by insured v. insurer, $200k CDs, $800k PDs
b. DP requires non-deferential tct & app ct review of jury PD awards
ii. Gore (US 1996)
a. fraud claim by BMW buyer v. mfr, $4,000 CDs, $2 mil. PDs
b. excessive PDs violate s DP rights; excessiveness assessed by:
(1) reprehensibility of s acts; (2) ratio of PDs to CDs (< 10:1
strongly preferred); (3) civil, criminal penalties for similar acts
iii. Williams (US 2007)
a. fraud claim by smoker v. tobacco co., $800k CDs, $79 mil. PDs
b. s attorney cannot ask jury to multiply PD award by # of nonparty victims; but jury can consider evidence of similar torts by
in assessing reprehensibility (?)
11.Menlove under Mathias

a. Consciously careless
12.D wealth with PDs
a. Courts have uniformly deemed evidence of a Ds wealth to be relevant to the
jurys determination of the size of any punitive award
i. Not because deserve greater punishment but because the actors
wealth may have factored into its decision to behave as it did
b. To reduce bias courts, bifurcate trials into Compensatory damages and PF case
and then separate for PD at which evidence of wealth is introduced
c. Trifurcate in some instances 1. Liability 2. CDs 3. PDs
13.Mathias and Antisocial conduct
a. Posner: suggests PDs assist in deterring and punishing criminal conduct that
would otherwise escape sanction due to detection or victim lacks incentive in
light of psychic and economic costs (i.e. scorched earth defense tactics)
14.Who Pays?
a. Vicarious liability: an actor other than the tortfeasor is held liable for the
damages that the tortfeasor inflicted:
i. Respondeat superior: when holding employers responsible for the
actions/wrongs of their employees, when employer has not done
anything wrong
a. Directly liable: if employer has done something wrong (negligent
in hiring/supervision)
b. Vicarious liability on the basis of relationship between employer
and employee; employee has to have been acting within the scope
of his employment
i. Need managerial capacity or act of company condoning
behavior
c. Both direct and vicarious liability can be pursued simultaneously
(which P should probably do to be on the safe side)
15.Exxon
a. Allows for PDs through vicarious liability, but approved instruction allowing
the jury to reduce the amound of any punitive award agains Exxon if it found
that corporate policy makers did not actually participate in or ratify the
wrongful conduct
b. Different than Mathias
i. With Mathias people injured through period of time
ii. Enough incentive to sue Exxon already and less barriers to suit
16.Insurance
a. Courts split on whether insurer can issue liability insurance policies
i. Reason for barring such policies: against public policy to let
perpetrators of egregious wrongs escape punishment

F. COMPARATIVE FAULT
1. Contributory negligence traditionally meant that if plaintiff was found to be at all at
fault, he couldnt recover reflected the view that if plaintiff wasnt coming to court
with clean hands, he was barred from recovery
a. Law now allows for recovery even when plaintiff is found to have been
party at fault
b. Rule of contributory negligence does not extend to claims of recklessness or
intentional wrongdoing on the party of the D
i. Thus carelessness by the P is irrelevant in suits for battery of false
imprisonment
ii. Also reflects shift away from thinking about tort law as corrective
justice growing concern with deterrence, using tort law as a way of
regulating risk-taking behavior in society
2. History of Contributory Negligence
a. Allowing D to walk free because P behavior is seen as a superseding cause
1) A reasonable person would have acted differently under the same or
similar circumstances
o OR P did no comply with a statute enacted for her own protection
(per se)
2) Ps unreasonable conduct was a cause of Ps injury
b. Smith v. Smith (Mass. 1824)
i. s careless wood-stacking caused s injury,
a. but s careless driving also caused s injury
ii. Contributory Neg
a. when & are both at fault for s injury, loses
b. CN = complete defense to negligence (not I.T.s)
c. Courts find a way to relax this over time and allow cases to go forward even in
presence of contributory negligence
i. Jury nullification: make it equitable decision, jury had enormous
discretion a judge is authorized to grant either a motion for JNOV or
motion for a new trial
ii. Last Clear Chance: a negligent plaintiff can nonetheless recover if
he is able to show defendant had the last opportunity to avoid the
accident.
iii. Emergency Doctrine
3. Historically, contributory negligence was a complete bar to recovery
a. Exception: admiralty law divided damages equally, regardless of degrees of
fault

i. Discarded in U.S. v. Reliable Transfer Ps tanker ended up on a


sand bar, in part because D failed to maintain a flashing light
a. Coast Guard was 25% liable, so they had to pay 25% rather than
50%
4. Contributory Negligence eliminated during period of 1970-1990 and replaced by
comparative responsibility in every state except:
a. DC, Alabama, Maryland, NC, and VA
b. However, all do away with the idea that any degree of carelessness on the part
of the P constitutes a per se bar to recovery
5. COMPARATIVE FAULT
a. Some at-fault Ps can recover, but the compensatory damages awarded those who
do recover are reduced in proportion to fault
b. must prove was at fault and that s fault was an actual cause of s
injury
c. Basic concept of CF is jury allocation of fault
d. Degree of culpability, not % causal contribution of their behavior
i. Jury looks at:
a. (1) How unreasonable was the conduct of the parties?
b. (2) What was the extent to which parties' conduct failed to meet
standards?
c. (3) Each person's abilities/disabilities
d. (4) Intent of each party
e. (5) Circumstances surrounding the conduct
e. Ps fault is assessed using objective standard where D must prove that Ps acts
were actual/proximate cause of Ps injuries
f. Policy for switching
i. Fairness/compensatory rationale (strongest), BUT tort law is not
efficient
ii. Economic: reduce # of accidents and costs (contributory negligence
under-deters, especially when Ds can predict when Ps arent taking
care)
iii. BUT could reduce the incentives of Ps to take care (though still
usually have a bar of more than 50%, so still have to be careful)
iv. BUT costs more to administer (have to allocate costs every time)
-SIDE NOTE: where comparative negligence, should stop taking subjective
characteristics of P into account (mental impairedness, religion, etc.) R3d
takes this stance
g. Different Regimes
Pure comparative responsibility: P can recover even if up to 99% liable for
own injuries (recovering 1%)

Each parties resoponsibility is assigned and P can recover when


any % is assigned to D
i.e. Florida, Disney deemed 1% at fault and force to bare 100%
of damages
Modified comparative responsibility: There is a threshold of fault above
which P is barred from recovery:
If P is 50% or more responsible, cannot recover
If P is more than 50% responsible, cannot recover
If P is slightly responsible they cannot recover
o Almost contrib neg.
Followed by 2/3 of Comparative Fault States
In cases in which there are multiple Ds, aggregate Ds fault and P
would have to be less liable than combined Ds
Justification: Concern for bait and switch
o Important feature of negligence law to require jury to make
comparison of fault and when evidence tilts one way want to
limit recovery
Hunt v. Ohio (1997): P is inmate who was instructed in use of snow blower
but not danger of sticking her hand in it; P puts her hand in snow blower to
clean it and loses fingers
P wins, but P 40% comparatively responsible for failing to use
common sense
Damages reduced under MCF by 40%

6. Joint and Several Liability


a. Traditionally available when two tortfeasors conspired together or acted in

concert
b. Also regularly applied when two negligent actors, acting independently of each
other, caused a single indivisible harm to the plaintiff (i.e. Ravo)
c. Also applied in hazardous waste contexts
d. Under Several liability, each D is ONLY liable for the share of responsibility
which he is allocated
e. Under Joint and Several liability, any one of the Ds can be made to pay 100%
of the damages (if other insolvent, etc.); responsibility for liability is shared
f. Elements
i. 1) when Ds share responsibility due to vicarious liability (for
example, respondeat superior)
ii. 2) when Ds act in concert - toward a coordinated purpose (drag race)
iii. 3) in situation where injury is indivisible ex: Ravo v. Rogatnik
g. Allocation of FAULT allocation of liability for damages
h. CONCERN/PRACTICALITY: Does this mean one minor tortfeasor will have
to shoulder all the burden? NO: Start with allocating share of responsibility to
Ds, and P can recover from either, and THEN D can bring restitution action
for contribution to get reimbursed for share of damages other D should have
paid.
i. State efforts to cut back on joint/several liability
i. 14 states have abolished JSL;
ii. 28 others have limited JSL
Some jurisdictions have abolished altogether or for certain cases
Concern about use of joint/several to seek out deep pockets only marginally
involved (municipalities, etc.)
A dozen states have abolished
A dozen have abolished J&S where the D is less than a certain % (typically
50%) at fault
A few states retain J&S for economic damages (like medical expenses) but
not for non-economic damages (loss of consortium, pain and suffering)
A handful retain the doctrine only where the P is not at fault
A handful retain J&S but reallocate the % share of any insolvent D for other
parties in the case (according to shares of fault)
Tennessee:
If multiple defendants are found liable in a civil action governed by
comparative fault, each defendant is only liable for the percentage of
fault allocated.
No defendant will be held jointly liable for any damages; however, the
doctrine of joint and several liability will still apply in civil conspiracy

cases and in product liability actions based on strict liability and


breach of warranty (among manufacturers).
j. Ravo v. Rogatnick (P 567, 1987): obstetrician and pediatrician work together to
result in brain injury, OB 80% liable, pediatrician 20%, but brain injury is
indivisible.
i. HELD: tortfeasors need not be acting concurrently or in concert to be
held jointly liable; if cause of injury can be easily divided up, it can be
several, if not must be joint/several.
ii. Jury apportionment of fault/negligence isnt the same as apportioning
responsibility
iii. Responsibility for injury cant be apportioned because indivisible
iv. Each act was a but for cause
v. With Comparative Apportionment relies on establishment of
causation (R3)
vi. Here use concurrent negligence: carelessness of each D was
necessary for Ps injury
vii.
Indivisible harm: two negligent actors, independently of each
other cause a harm where there was no way to tell which tortfeasor
caused which portion
k. Bencivenga v. J.J.A.M.M. (P. 574, 1992) - P assaulted by unknown person in
nightclub while Ds employee bouncers watched and did nothing.
i. NJ court says cannot allocate damages to absent (fictional) D.
a. Policy reasons
i. Def has greater incentive to join other potential defs
ii. Club also in better position to know assailants identity
ii. BUT you CAN apportion damages between negligent and intentional
tortfeasorseven when negligent takes whole burden b/c of joint and
several liability.
iii. NJ Approach: Hybrid that authorizes JSL in indivisible harms cases
but only against those tortfeasors whose percentage of responsibility
surpasses 60%, then P can recover 100%
7. Comparative fault v. Comparative Apportionment
a. Criticism says comparing apples and oranges
b. Courts more willing to throw things like intentional torts into the comparison
with CF transistion
c. Two problems:
i. Joint and several liability if one D is judgment proof
ii. Phantom defendent
d. Comparing Causation with apportionment
i. Not about wrong doing

e. comparing acts with CF


f. Equitable approaches
i. Pro Rata: before CF if D1 forced to pay 100% of damages and sued
D2 he could only get 50% regardless of fault
ii. Pro Tanto: Reduces non settling D liability by amount paid by
seetling D
g. R3 has taken the opposite tack by using the term apportionment to encompass
both plaintiff-versus-defendant and the defendant-versus-defendant
comparisons.
G. DEFENSES
1. Assumption of the risk
a. Express Assumption of the Risk
i. agreement between & ; agrees in advance to waive right to sue
for injury caused by s wrongdoing
ii. Argument that victim is barred from recovering either because she
has agreed to relinquish her right to sue because P knowingly and
voluntarily took on the risk that she might be injured by careless
conduct on the arty of the defendat(s)
iii. Deals with whether some sort of written waiver/release that plaintiff
has signed allows def to establish that plaintiff forfeited his right to
sue
iv. In Jones and Dalury the waiver is written but express assumption
of risk may also be invoked based on implied contract
v. Can be invoked as a defense plaintiff assumed the risk, and is
barred from recovering anything
ii. Jones v. Dressel (p 452, 1981): Skydiving plane crashes and P had
signed a waiver, and court enforce the waiver
a. Takes four corners approach
b. There was a buy back provision
c. Not actually doing the activity when it occurred
iii. Dalury v. Ski LTD. (p P 458, 1995):P was a skier injured when it hit a
pole on the slopes
a. Waiver not enforce-def not allowed to argue P assumed the risk
b. Two make adhesion contract (K drafted unilaterally by a business
enterprise and forced upon an unwilling and often unknowing
public for services that cannot readily be obtained elsewhere,
generally not bargained for, but imposed on the public for a
necessary service on a take or leave it as is) must show parties

were greatly disparate in bargaining power, no opportunity for


negotiation, or that services could not be obtained elsewhere
c. 4 factors whether exculpatory agreement factors considered Tunkl
(not all are necessary)
i. Duty to public
ii. Nature of the service performed
iii. Whether the contract was fairly entered into
iv. Whether the intention of the parties is expressed in clear and
unambiguous language
d. Vs Jones:Sky diving not a service of great importance to the
public, did not possess advantage of bargaining strength over
Jones, and the contract was not an adhesion contract
e. Test for common carrier: whether the carriage by air is merely
incidental to the person's other business or is, in itself, a major
enterprise for profit
iv. Central issue in Jones and Dalury are the waivers void as against
public policy?
a. Doctrinal questions to consider
i. Does the waiver actually cover the particular risk that
ensued?
ii. Was the contract knowingly and voluntarily entered into?
v. Court in Jones and Dalury apply the same multi-factor balancing
test (from Tunkl case)
a. Concerns a business of a type generally thought suitable for public
regulation
b. Party seeking exculpation is engaged in performing a service of
great importance to public, which is often a matter of practical
necessity for some members of the public
c. Party holds himself out as willing to perform this service for any
member of public who seeks it
d. As a result of the essential nature of the service, in the economic
setting of the transaction, the party invoking exculpation possesses
a decisive advantage of bargaining strength against an member of
the public who seeks his services
e. In exercising a superior bargaining power, the party confronts the
public with a standardized adhesion contract of exculpation, and
makes no provision whereby purchaser may pay additional
reasonable fees and obtain protection against negligence

f. As a result of the transaction, the person/property of purchaser is


placed under sellers control, subject to the risk of carelessness by
seller or his agents
vi. Outcome is different in the cases because
a. In Jones, plaintiff had the option of accepting the waiver or paying
an additional $50 to retain the right to sue
i. Dalury did not have such an option
b. In Dalury, more people touched by the type of business/service
provided, also policy rationale places responsibility for
maintenance of the land on those who own or control it. Property
owner alone can maintain and inspect their premises.
i. If D's were permitted to obtain broad waivers of their
liability, an important incentive for ski areas to manage risk
would be removed, with the public bearing the cost of the
resulting injuries
c. Moreover, reliance on the private nature of D's property would be
inconsistent with societal expectations about privately owned
facilities that are open to the general public
d. Could also try to rationalize using the lowest cost-avoider
determination
i. Ski resort may be better equipped to insure against accidents
e. Control: If we can identify exclusive control over risk that caused
harm courts are reluctant to enforce these agreements outside of
extreme sports
i. Jones had lack of exclusive control
ii. Dalury had exclusive control so even with Tunkl might turn
out different
vii.
Berry v. Greater Park City Co:
a. S.Ct. Utah reasoned that the policy considerations disfavoring
enforcement of waivers of negligence liability for injuries arising
out of recreational skiing do not apply to a kind of skiing that
involves only a "limited group of expert , competitive skiers"
viii.
Benefits of K
a. Allow allocation of risk
b. Consistently honors shifting risk
c. Allows ability to enjoy greater range of activities
i. Dalury elevates safety value over freedom of K
b. Implied Assumption of the Risk

i. no agreement, but freely and knowingly chooses to encounter a


danger posed by s carelessness
ii. Deals with Ps conduct
iii. Modern trend away from being a complete defense
Implied non-contracted waiver of liability, D must show that Ps actions and
behavior indicate P implicitly assumed risk
D must show that:
(1) P knew about risk (cannot be implied)
Actual knowledge: subjectively knew and appreciate
magnitude/implications
(2) P voluntarily assumed risk
Negated by force, fraud or necessity (no reasonable altrnative)
(3) P actually encountered the exact risk which assumed liability for
Implied assumption states:
1) Complete bar to recovery (contributory negligence regime)
2) Reasonable v. unreasonable (comparative negligence regime)
Reasonable - complete bar
Unreasonable- comparative negligence
3) Nonexistent in some states (only can do comparative negligence)
4) NY Hybrid Statute: implied assumption of risk as distinct from
comparative, BUT only reduces award by %age (partial defense)
Other states: primary and secondary implied assumption of risk (Knight Approach,
Most Common)
Primary-- actually NOT assumption of risk, but rather NO duty (risks inherent
in activity, doesnt matter whether P actually understood risks)
Complete bar to recovery, even if P is not subjectively aware
Not a defense but a no duty rule
Two crucial differences between this and Smollet
1. As a no duty doctrine this is a question of Law for judge
2. Not subjective about whether P knew the risks, if Ps
involvement was voluntary and risks were inherent the
doctrine can still apply
Ex: recreational sports
P knew risk because its open and obvious-- the more obvious the risk;
the less likely a breach will be found
Murphy v. Steeplechase Amusement Co (NY 1929)CARDOZO: p gets on
a ride where he runs on a conveyor belt and then falls; he breaks his

kneecaps; he had been watching the ride and knew what it was/did,
Flopper case
P had assumed the risk (open/obvious), no breach
Secondary - there is an actual implied assumption of risk (but isnt this just
comparative fault?); can be broken into reasonable and unreasonable
Folds decision into CF, not a complete bar
For courts that adopt Knight, Smollett defense would fall here
Q: has P voluntarily and knowingly encountered the risk
c. Smollet v. Skayting Dev. Corp (p. 463, 1986)
i. Issue: Virgin Islands have shifted towards comparative responsibility
does implied assumption of risk still exist as a doctrine?
a. Law doesnt explicitly state that it was replaced
b. Restatement 2nd 343A provides that a possessor of land isnt
liable to invitees for harm caused to them by any activity or
condition of the land whose danger is known or obvious to them
i. Requires plaintiffs implicit acquiescence to known or
obvious dangers
ii. Holding: implied assumption of risk is still a doctrine in VI up to
legislature to eliminate it
a. Says P can be seen to have impliedly assumed the risk because of
knowledge
iii. Assumption of risk can still be applied to non negligent conduct
which constitutes waiver or consent but which involved no negligence
a. Here the absolute bar still remains
iv. Keegan/Smollet sets up weird tension
a. Two tiered approach
i. P acting unreasonably
ii. Knowledge of risk merges to CF
i. Here P who is acting reasonable is punished more than an
unreasonable P
rd
v. 3 Restatement is at odds with the outcome says that when the
plaintiff has been negligent, those cases should be dealt with under the
comparative fault regime
a. Gets rid of implied assumption of risk but leaves a little room for
dealing with some cases that straddle the boundaries between
assumption of risk and comparative fault under the rubric of
express assumption of risk

d.

e.

f.
g.

i. Many jurisdictions have also stated that move towards


comparative fault regime displaces the doctrine of implied
assumption of risk
ii. With merger only relevance of IAOR would be duty to
mitigate damages
Duty to Mitigate Damages
i. Even if you find no fault, still duty to mitigate damages and damages
will be reduced if you dont
a. I.e. motorcyclists failing to wear a helmet, seatbelts
i. Assuming no statute requiring this
ii. Ps conduct made damages worse
i. Secondary Assumption of the risk
Baseball Stadiums and other venues
i. Distinction from inherent risk from those that are not
a. I.e. Coomer: MO S.Ct says negligence action can be pursued when
hit by a hot dog
STATUTES OF LIMITATIONS and REPOSE
i. Statute of Repose: triggered at time of action
ii. Statute of limitation: triggered with injury
Old Immunities Modified by statute
i. Spousal
a. common law
i. domestic abuse a private matter
ii. allowing negligence claims enables collusive litigation
b. by 1970s, most states reject blanket spousal immunity
c. but see Heinze (Neb. 2007) (husband v. wife suit barred by guest
statute)
ii. Charitable
a. immunity for charitable orgs. has been modified in most states
i. charitable hospitals now liable for MDs negligence (resp.
superior), but special liability limits sometimes still apply
iii. Parental
a. limited or abolished
i. worries remain that parents
and children might collude
b. privilege to discipline exception

H. STATUTES
1. Role of Statutes
a. creating new torts

i. NY Civ. Rights. L. 50-51 makes privacy invasions actionable


b. eliminating common law torts
i. OH statute abolishing heartbalm actions (Mussivand, Ch. 2)
c. replacing tort law with alternative schemes
i. Workers Compensation (White, Ch. 11)
d. altering remedies for successful claimants
i. Damages caps (Ch. 8)
e. abolishing defenses
i. Federal Tort Claims Act (Riley, Ch. 7)
f. changing procedural, evidentiary rules
i. Statutes of Limitations (Cole, Ch. 9)
2. NEGLIGENCE PER SE
a. Elements
i. Clear statutory duty
a. Did D Violate statute without legally acceptable excuse? (Breach)
b. Was the violation an actual cause of Ps injury (Actual Cause)
ii. Death or injury must result from an occurrence which the statute was
designed to prevent ( Is it a safety statute, Scope of Risk analysis)
a. Duty as Cardozo illustrates in Palsgraf
iii. Person suffering must be from the class of persons for whose
protection the statute was adopted.(Duty)
a. Must answer yes to all of these
b. If NO P cant use NPS, but can introduce elements as evidence to
jury and jury has full discretion on weight given to the statute
b. Why have negligence per se?
i. Promotes enforcement of statutes
ii. Avoids inconsistency between statutes and common law
iii. Statute more democratic than common law
iv. Uniformityjurors wont overrule a commonly followed statute
v. Statutes come from democratically-elected legislatures
a. BUT ordinary neg would be determined by jury of lay people
(BUT legislature prob more representative than small jury)
vi. Statutes as forward-looking (technology-forcing) informed, expert
view
vii.
Jurors feel more comfortable using expert views.
viii.
Statutes are more predictable/transparent than decisions from
judiciary (juries) published, widely available
ix. Reduces litigation costs (juries dont have to establish standard for
every case, less experts to pay for), but in reality.

c.

d.

e.

f.

x. Reduces enforcement costs for govt (private attorney generals)


xi. Institutional comity: want laws between legislature and judiciary to be
consistent
a. BUT juries can set higher standards than statutes (unless
legislation says otherwise)
ONE APPROACH
i. R3 14 requirements (a) a violation of statute, (b) statute intended to
protect against the type of accident and (c) the class of which victim
was a member
TRADITIONAL APPROACH
i. Martin v. Herzog (p. 74) Cardozo held that P could sue under a
statute requiring use of headlights
ii. Not just a prima face burden shift; violation of statute IS negligence
iii. Safety statute violation is negligence
Dalal v. City of New York (p. 384, 1999): D not wearing glasses at time of car
accident even though she needed them.
i. Violation of NY Traffic Law
a. DL only relates to authority for operating a vehicle and thus not a
safety statute
b. BUT wearing glasses is required and does constitute a safety
statute
Bayne v Todd Shipyards Corp (P. 385, 1977): P (non-employee) unloading
goods from Ds loading platform (no guardrails); P fell and got injured
i. P recovered because this was neg per se: P was in the class of Ps
(statute broadly protected workers, did not exclude 3rd party
contractors like P) that was meant to be protected from this type of
injury.
ii. It does NOT matter that it was an administrative regulation instead
of a statutory one b/c:
a. Expertly created (probably even more than statutes)
b. Like statutes: community input, power from Legislature
c. Predictable, transparent
d. Reduces litigation and enforcement costs
e. Institutional comity
iii. BUT (dissent in Bayne v. Todd Shipyards Corp):
a. Too many regulations (overwhelming), not always
necessary/important
b. Should just be evidence, not per se standard!
c. Not actually democratically created (BIG diff from statutes)

g.

h.
i.

j.

k.

l.

m.

iv. Issues figuring out intent of regulators (unlike legislature), unless fed
level (have to publish reasons)
v. NOT all states make regs per se (NY, for example)
even w/ these preconditions met, NPS claims have elements comparable to
common law negligence
i. R2 Four Factors
a. is among those whom the legislature or agency
intended to protect by enacting the statute or regulation? (a+b)
b. was the accident of a type the legislature or agency
meant to discourage by enacting the statute or regulation? (c+d)
ii. Framing issue (type of harm and person meant to protect) is a matter
of law for the judge
Gorris v. Scott ship owner failed to erect pens for sheep, violating Contagious
Disease Act. Storm washes sheep over. D not liable because statute was not
intended to protect against type of injury
Victor v. Hedges (p 388, 1999): D parks car on sidewalk to show P his stereo
which is against CA vehicle code. P gets hit and sues tries to invoke NPS
i. P was among protected class of pedestrians
ii. BUT not the type of accident legislature was trying to prevent
Regulations
i. Compliance sets the floor but not the ceiling
a. Evidence of non-negligence but does not preclude a finding that
the actor is negligent
b. Could be other reasons for negligence
Used as a Defense?
i. Probably not, statute is one element but dont set standard for
reasonable care
ii. Statutes set floor
iii. NPS with compliance doesnt work in converse
a. Most court will allow this as evidence for jury
Judge and Jury
i. Judge:
a. Decides whether law is a safety statute
b. Jury instruction
i. Violation of statute is negligence if it is excused or not
ii. Jury:
a. Causation
i. Not only about violation but also if P contributed
b. Damages
CARDOZO Rationale for NPS

i. Legislative supremacy
ii. Constraining jury not judges
iii. Judge in fact deciding force of statute
n. Implied right of action
i. statute imposes a duty apart from the duty-not-toinjure-through-carelessness recognized by the tort of
negligence
ii. statute is silent as to whether violation gives rise to a
claim for damages by a person injured by a violation
iii. injured by s violation sues for breach of statutory duty
iv. Q: does the statute implicitly create a private right of action, such that
is entitled to redress from for s violation of the statute?
a. Courts hesitant to create COA here
EXCUSE
I. EMOTIONAL HARM
1. Negligent Infliction of Emotional Distress (NIED)
2. Wyman v. Leavitt (p 748, 1880) No Injury rule
a. Issue: rocks carelessly blasted onto Wymans property Mrs. Wyman alleges
she suffered anxiety as a result of the blasting, wants to recover
b. Holding: Mrs. Wyman cant recover
c. Court says her emotional distress claim isnt parasitic to the property damage
her husband owns the property (not her), and also reflects traditional reluctance
to allow recovery for emotional distress claims parasitic to claim for negligent
property damage
i. If damage to property had been intentional, or if she had owned the
property, may have been able to recover
d. Development of narrow rules allowing plaintiffs to recover for NIED,
independent of physical injury
i. Physical impact rule allowed recovery for mental distress if
plaintiff could show he had suffered a contemporaneous physical
impact
a. Watered down so the impact merely had to be slight or trivial, i.e.
dust touching his skin
3. Robb v. Pennsylvania RR (p 749, 1965): Zone of Danger (ZOD)
a. Issue: plaintiffs car was stuck in a rut negligently caused by def at a RR
crossing she jumped out right before the train hit and destroyed her car and
claimed NIED

b. Holding: plaintiff is allowed to recover for her mental distress suffered from
fearing for her physical safety
c. Court moves away from physical impact rule adopts zone of danger test
d. Takes approach that it is their duty to afford a remedy and redress for every
substantial wrong (echoed in Dillon) not concerned about unlimited liability
or trivial claims
e. Zone of danger test requires 3 elements for plaintiff to recover for mental
distress damages:
i. Plaintiff must be within the zone of physical danger from defs
carelessness (objectively determined, not subjectively/from
plaintiffs perspective)
ii. s contemporaneous awareness causes her to fear for her own
safety, and
a. i.e. cant be asleep
iii. Plaintiffs fear of being physically injured must be the cause of
her fright / emotional distress
a. Most jurisdictions dont even go here/dropped this requirement
f. ZOD Arbitrariness?
4. Under Either Rule Still Can Recover:
a. Intentional Torts
b. Distress is parasitic on physical harm
c. Special relationships
d. Bystander liability
5. R3:
a. places the other in immediate danger of bodily harm and the emotional
disturbance results from the danger; or
b. occurs in the course of specified categories of activities, undertakings, or
relationships in which negligent conduct is especially likely to cause serious
emotional disturbance.
6. Special Relationships
a. Just because contract provides sole COA to sue does not mean the P can recover
for ED
b. special-special relationships
i. Those who deal with issues surrounding death
a. Mortician duty to family to not mishandle bodies
ii. Passengers of Common Carriers
a. Duty with pure ED
iii. Employer-Employee must be special
a. Hedgepeth: Dr treating someone for HIV, says he is HIV positive
and he finds out 5 years later he is negative

b. Rowell: Client presented attny with proof of his innocence and


lawyer delayed in presenting it for 10 days, meanwhile he stayed in
jail.
iv. P objectively vulnerable
a. P relied on D
b. A symmetry of power
c. Children in daycare, duty owed for pure ED
7. Bystander Emotional Distress
a. Waube v. Warrington (p 778, 1935):
i. Issue: mother suffers emotional distress from seeing her child run
over; she dies and her husband brings an NIED suit
ii. Holding: court denies recovery because it says the mother wasnt
within the zone of danger
a. Defs duty / plaintiffs right to recover cant be justly extended to
include recovery for physical injuries sustained by one outside of
the range of ordinary physical peril as a result of the shock of
witnessing anothers danger
b. Invokes Palsgraf def doesnt owe a duty to anybody in the
world, but only to those who have a legally protected right
b. Dillon v. Legg (p 781, 1968):
i. Issue: Mom walking with daughters saw one get run down. Under
ZOD daughter who witnessed her sister get killed can recover, but she
was out of the zone of danger
a. Under AMAYA=No Recovery
i. CARDOZO doesnt think Amaya duty is arbritrary
ii. Holding: Recovery for emotional distress claims can be properly
limited by determining whether a defendant owes a plaintiff a duty of
care, and whether this duty is breached
iii. TEST :
a. Whether P was located near the scene of an accident as opposed
to some distance from it
b. Whether the shock alleged by the plaintiff resulted from his
sensory and contemporaneous observance of the accident
c. Whether the plaintiff and the victim were closely related, as
contrasted with an absence of any /distant relationship
iv. Strong endorsement of Andrews approach= duty owed to everyone
a. BUT, If contributory negligence found then P cannot recover
i. CN of another party could bar Ps ability to recover
ii. TENSION with Andrews

c. Thing v. La Chusa (1989)--Automobile strikes child, mother arrives after being


told that child was struck, sees mangled child and believes him to be dead; sues
for NIED.
i. Ct. finds temporal and spatial proximities to moment of injury NOT
met; narrows Dillon holding by turning guidelines for consideration
into requirements.
d. DUTY v. PROX CAUSE
i. With Dillon Test part of what is going on here might be a prox cause,
but court clearly taking a strong duty approach
ii. Androzo II
a. All things considered
b. Public policy concerns
c. Foreseeability
d. Duty encompasses a lot of prox cause argument
e. 29 jurisdictions take Dillon test and R3
i. Next largest set take ZOD
ii. Small amount of jurisdictions have collapsed duty based on same
analysis for physical harm
a. Tennessee and Montana
i. Camper
8. Intentional Infliction of Emotional Distress (IIED)
a. Basics
i. Imposes liability on an actor who, by means of outrageous conduct,
intentionally or recklessly causes severe emotional distress to another
a. Came into existence as cause of action only in the mid twentiethcentury
ii. Examples before IIED
a. Wilkinson: man falsely informs spouse her husband has been in an
accident, without specifying CoA on which P could rely court finds
for P
b. Nickerson: vicious practical joke about family treasure
iii. Prosser (1939) R1(1948)
iv. Courts had come to articulate a new tort cause of action. Under which
an actor could be held liable for conduct:
a. That is outrageous
b. That is undertaken for the purpose of causing the victim
emotional distress so sever that it could be expected to affect
adversely his physical health; (INTENT) and

c. That causes such distress (even if this does not generate


physical harm)
i. IIED not available for mere insults, indignities, threats,
annoyances, petty oppressions, or other trivialities.
ii.
v. -IIED has taken root in US tort law and widely recognized
vi. Although often pleaded, these claims rarely succeed
b. Problem for Plaintiff with IIED
i. 1-yr limit for claims of assault, battery, f.i. expire before suit is filed
ii. IIED claims, governed by 3-yr period for unspecified torts, are not
time-barred
iii. but because IIED is a gap-filler, can only benefit from 3-yr period if
there is conduct that is IIED but is not battery, assault or FI
iv. Expressio Unius: a principle in statutory construction: when one or
more things of a class are expressly mentioned others of the same
class are excluded
c. Recklessness with IIED
i. Hunt v. State (P 711, 2013): Student brings action against school
resource officer because Ds interrogation tactics
ii. Issue: Did P meet requirements to survive SJ?
iii. Holding: IIED is reinstated
a. Recklessness (as to risk of traumatizing victim) suffices to
establish the intent element of IIED
b. R3 Phys. & Em. Harm 2
i. A person acts recklessly if: [he] knows of the risk of
harm or knows facts that make the risk obvious , and the
precaution that would eliminate or reduce the risk involves
burdens that are so slight as to render the failure to
adopt the precaution a demon-stration of [his] indifference to
the risk.
ii.
c. R2 46(2): Conduct directed at and reckless as to s emotional
well-being
i. Where . . . [outrageous] conduct is directed at a third
person, the actor is subject to liability if he recklessly
causes severe emotional distress
i. to a member of such persons immediate family who is
present at the time . . . , or
ii. to any person who is present at the time if such distress
results in bodily harm.

d.

e.

f.
g.

h.

d. R3 rejects directed at requirement and extends liability


beyond 46(2)
i. Doe: P alleged having been sexually abused by priest and
said church failed to take steps to notify authorities, not
directed at
e. With recklessness it distinguishes IIEd from the other intentional
torts with which it is sometimes grouped because not actual intent
there
Judge and Jury
i. Many courts find extreme outrageous is question of law for the
court
a. Hunt
ii. Other deem it for the jury
iii. R3: intermediate position deeming the question one of the fact for the
jury but also stating that courts should play a more substantial
screening role (gate keeping) because malleability of extreme and
outrageous standard and the higher likelihood of biased jurors
High bar of outrageousness
i. Must be more than one shot proposition
a. See Jones v. Clinton
ii. Need specific act + ongoing acts, something more
Proof of Emotional Distress
i. IIED P must prove that the conduct has caused her severe emotional
distress
Intent or Harm Based Tort?
i. Depending on how it is framed can be seen as either
a. Egregious way in which the D mistreated the P?
i. Intent
b. Harm caused as a result
i. Harm
Transferred Intent and Bystander Claims
i. R2:
a. (2) Where[extreme and outrageous] conduct is directed at a
third person, the actor is subject to liability if he intentionally or
recklessly causes severe emotional distress
b. (a)to a member of such persons immediate family who is present
at the time, or
c. (b)to any other person who is present at the time, if such distress
results in bodily harm.
i. Important restraint, Hunt probably cant recover

ii. Suggesting special doctrine of transferred intent


a. If establish intent against AB, intent transferred in limited
circumstances
i. Prima Facie Tort
i. Recognition of special cause of action designed to enable victims to
pursue claims for intentional wrongs that continue to fall through
doctrinal cracks
ii. New York formal elements of such a claim:
a. (1) intentional infliction of harm;
i. Intent requires not only proof of intent to inure, but in
additional proof that the actor formed this intent solely out
of a "malevolent" desire to injure the P
ii. I.e. if D injures P for the purpose of advancing self-interest,
not actionable in PFT because D has acted out of desire to
enrich himself
b. (2) resulting in special damages (i.e., discrete and measurable
monetary losses);
c. (3) without excuse or justification;
d. (4) by an act or series of acts that would otherwise be lawful.
j. IIED Defenses and Constitutional Limits
i. In principle, standard defenses apply
a. e.g., consent; self-defense
ii. But . . .
a. facts establishing defenses tend to show failure of s p.f. case
(e.g., if consents, s conduct probably was not outrageous)
iii. First Amendment Limits
a. Public figures: Hustler v. Falwell (1988)
i. sues for crude, upsetting ad parody
ii. SCT: vagueness of outrageousness creates an
unconstitutional risk that juries will use IIED to punish
speech just because they find it distasteful
b. Private figures: Snyder v. Phelps (2011)
i. sues for disruption of sons funeral
ii. SCT: unconstl to impose liability for speech just because
society deems it offensive
J. STRICT LIABILITY
1. Negligence vs. Strict Liability

Fairness

Incentives/
Deterrence

Legal
Process

Negligence

Strict Liability

Its not fair to hold someone


liable for an accident if theyve
taken care not to cause it

As between an innocent injurer and


an innocent victim, it is not fair to
leave the victim to bear the loss

SL too strongly discourages


productive activity; it will
induce people not to drive, etc.

Negligence induces care in activities


(drive carefully!), but SL further
induces us to avoid or limit risky
activities (dont drive, or drive less!)

Judges and jurors can be


relied on to determine if an
actor was careless; the cost of
making this determination is
worth it

It is difficult to determine what


counts as carelessness; negligence
introduces uncertainty into law and
generates excessive litigation costs

2. Property Torts (Trespass to Land PF case)


a. Actor (A) causes a physical invasion of property lawfully possessed by another
i. (invasion can be in person or by animals, devices or substances for
which the actor is responsible) and
b. A intended to make contact with the property invaded
i. (A need not know, or have reason to know, that the property is in the
possession of another)
ii. trespass to land combines an intent requirement with a form of strict
(no fault) liability
3. Trespass to Chattel
a. R2d Torts, 222A(1)
i. Conversion is an intentional exercise of dominion or control over a
chattel which so seriously interferes w/ the right of another to control
it that the actor may justly be required to pay the other its full value
b. R2d Torts, 218
i. One who commits a trespass to a chattel is subject to liability to the
possessor of the chattel if, but only if,
a. he dispossesses the other of the chattel, or
b. the chattel is impaired as to its condition, quality, or value, or
c. the possessor is deprived of the use of the chattel for a substantial
time, or

d. bodily harm is caused to the possessor, or harm is caused to some


person or thing in which the possessor has a legally protected
interest
i. Unlike trespass to land, no liability w/o actual harm to
property or deprivation of use
ii. As w/ trespass to land, liability can attach w/o fault (e.g.,
taking anothers umbrella rsbly believing it to be ones own)
c. Nuisance
i. nuisance unreasonable interference w/ anothers lawful use &
enjoyment of her land
ii. Like trespass, nuisance:
a. requires have a possessory interest in affected property
b. frequently involves requests for injunctive relief
c. doesnt require proof of physical damage to property
d. doesnt require proof of s failure to act reasonably
iii. Unlike trespass, nuisance:
a. intent to make contact w/ relevant phys. space not required
b. usually a continuing rather than one-off interference
c. requires unrsble interference with s use of her property
d. inquiry concerns the nature of the interference, not s conduct!
e. liability turns in part on the s interest in continuing
the allegedly offending activity
4. STRICT LIABILITY
LOOK AT SIX FACTOR TEST AND POLICY CONSIDERATIONS FOR OUTCOME
o 6 Factor test for Strict Liability - 2d Restatement of Torts
520:
a. Existence of high degree of risk (P)
Abnormally dangerous/ultra hazardous activities
b. Likelihood that harm resulting from risk will be great (L)
c. Inability to eliminate risk by exercise of reasonable care
(Negligence simply will NOT do)
d. Extent to which activity is not a matter of common usage
Uncommon usage means nonreciprocal risk: P unlikely to know how
to avoid risks D imposes, or even to be aware of such risks
e. Inappropriateness of activity to place where carried on
f. Value to community of activity is outweighed by its dangerous attributes
C and D end up being the most important in case law; dont need ALL
factors necessarily.

Strict liability = legal responsibility for damages or injury even if the D not at fault or
negligent
o ACTUAL/PROXIMATE CAUSATION is still an important factor (but NO need to
prove breach).
Ex: Blasting noise causes frightened mink mom kills babies see Foster v.
Preston Mill Co. (WA 1954).
Areas of Strict Liability
o 1. Abnormally dangerous activities: blasting, large reservoirs, gas/tanker trucks,
gathering/keeping things which are a nuisance if unleashed
o 2. Damages caused by wild animals
o 3. Some products liability
o 4. Respondeat superior
o RES IPSA LOQUITUR can be a way to basically impose strict liability.
Defenses to strict liability:
o Comparative negligence
o Assumption of risk (Ex: Indiana Harbor)
Justifications for strict liability

Economic rationale: reduce/change activity levels, for example WHERE


done (encouraging D to revisit how much they do activities, since courts are generally
reluctant to assess activity levels)

BUT strict liability doesnt encourage increased safety any more than
negligence

Reduces transaction costs by not requiring proof of fault

HOWEVER, activity levels should NOT be reduced when no readily


available substitutes AND activity does a public good (Adams v. Bullock,
above-ground trolleys) OR when D is not in the best position to make things
safer/lowest cost avoider (Indiana Harbor - homeowners should move)

Lowest cost avoider

Internalize costs of activity (society shouldnt have to pay); nonreciprocal risks


Distributional consideration - Want to impose costs on party who can spread them
most efficiently (deep pockets? But see Indiana Harbor)
BUT what about NON-TORT remedies like real INSURANCE?
Traynor in Escola - this SHOULD be strict liability
Compensatory rationale

Ps will generally be compensated more than negligence, but may get less
(jury lacks moral blame, smaller pie)
BUT administrative compensation scheme may be more efficient
Justice/fairness
Problems of proof: P may not have evidence (often b/c it explodes)
High probability that accidents of this type caused by negligence

Under-deterrence if evidence unclear/doesnt necessarily lead to


negligence verdict in court.
Parties imposing nonreciprocal risks upon others; should have to
internalize
Autonomy theory - should be compensated when autonomy is
interfered with
Against (Holmes)
o If about deterrence, why have strict liability?
D has not necessarily been negligent and thus is NOT being deterred from
any wrongful behavior
o Strict liability deters innovation
Makes companies not want to be first in anything (new liability)
o Goes against autonomy theory (restricting actions without good
reason)

a.
b. Negligence
i. Holmes v Cardozo
a. CARDOZO wins, courts dont decide, the jury typically will be
defined as what is expected
c. Strict Liability doing something different
i. Legal analysis doesnt proceed on a case by case basis
ii. Once we decide one activity is strict liability activity is always
strictly liable
iii. Once strict liability is appropriate
a. Legal issue not for jury
i. Jury can still decide cause, damages
ii. Takes away case by case analysis
5. Abnormally Dangerous Activities
a. Draft Third Restatement (20): an activity is abnormally dangerous and
subject to strict liability if:

b.

c.

d.
e.
f.

g.

i. The activity creates a foreseeable and highly significant risk of


physical harm even when reasonable care is exercised by all
actors, AND
ii. The activity is not of common usage
a. Endorsing Posner view that factors C & D should drive the
analysis, other factors inform these factors
True strict liability in tort
i. the intentional aspect of property torts is absent
ii. common law doctrine (not part of a tort replacement scheme such as
workers compensation)
Very limited in application
i. keeping of wild animals
ii. use of explosives, radioactive materials
iii. keeping of reservoirs
Courts have resisted extending this form of S.L. to other kinds of conduct
Doctrinal Origins
Rylands v. Fletcher (p 878, 1868): Reservoir breaks and floods coal mines,
damages Ps property
i. Cairns, L.C., liability is strict for injuries caused by non-natural uses
of property
ii. HIGH RISK OF DANGER/ESCAPE and leading to danger/mischief
iii. Property rights!
iv. Not negligence, not nuisance, not trespass
a. what counts as a non-natural use?
i. introduced onto the land by ?
ii. unusual, uncommon, out of place ?
iii. involves efforts to contain a thing prone to escape & do
damage ?
v. Cranworth: when one person, in managing his own affairs, causes,
however innocently, damage to another, it is obviously only just that
he should be the party to suffer.
Klein v. Pyrodyne Corp (P. 875, 1991): Fireworks explode near spectator and
injure P
i. Holding: Operator of a public fireworks display is strictly liable for
harm resulting therefrom
ii. Six factors (see above)
a. a, b, c: explosions
b. c: reasonable care possible?
c. d: common usage to watch, but NOT common usage to set off
d. e: fairgrounds are OK, actually (against imposing strict liability)

e. f: 4th of July is an important community activity, but SAFETY


first
f. Policy issues
i. Fairness: who should pay for damages? cheapest cost
avoider? Pyrodyne acted, P viewed (autonomy theory); all
evidence is blown up.
g. R3 endorse the application of comparative fault to abnormally
dangerous activities
h. Modern jurisdictions: If you have liability against manufacturer
and pyrotechnic
i. Case would go to jury for apportionment
ii. With Strict Liability =no fault so standard is comparative
causation
i. In future Pyrodyne type cases
i. Court determines whether activity is appropriate
ii. P still has to establish
i. Injury
ii. PC and AC
iii. What injuries are compensable
h. Limits
i. for injuries resulting from an UA, liability is strict, but
a. must be the right kind of victim (a bystander, not a
participant)
i. BYSTANDER not a participant
b. the injury must come about in the right manner
i. Foster v. Preston Mill Co. (Wash. 1954) (mink eating cubs
outside the scope of risk)
ii. Klein (fireworks mfrs possible negligence not a
superseding cause)
ii. moreover, defenses such as comparative fault & assumption of risk
will typically today apply on the same terms as they do for negligence
claims
a. CF can reduce damages of D
6. PRODUCTS LIABILITY
a. Contract Heritage
i. Warranty = an assurance or promise of quality
a. can be express or implied
ii. Pro-Consumer Aspect
a. warranty liability is strict, not fault-based
b. seller warrants quality, not mere rsble care as to quality

iii. Pro-Seller Aspects


a. source of duty is K privity often remains a limit
b. seller can try to disclaim all warranties (as is)
c. prior to suit, consumer may be reqd to give timely notice of
breach & oppty to cure
iv. Henningsen (NJ 1960): the MacPherson of warranty law
a. Expansion of K warrant
i. Not limited to purchasers extends to others
ii. Wty of fitness is implied w/ any sale of car
iii. Wty disclaimers, limits held void against pub. policy
iv. Wty extends to product users not in privity
b. Majority of jurisdictions abdandoned the privity requirement in
implied warranty actions
b. Prima face case
i. Injury
ii. Product sold by D
iii. D = a commercial seller of the product
iv. Product was defective at time of sale
v. Causation (actual, proximate)
a. 1 & 5 Activities and Products
b. 3,4,5, differ with products, most controversial
c. 2,3,4, main factors
d. Time of Sale
e. Must be used in the intended way
c. Escola v. Coca Cola (p. 895, 1944) Traynor concurrence; mfr should be
directly and strictly liable
i. Avoids privity by working within negligence law
ii. Concurrence (Traynor) - use of Res Ipsa disingenuous; were
imposing strict liability for products here and should just say it
a. Company is cheapest cost avoider
b. Manufacturer acted; put out product
c. Loss spreading
d. Judicial efficiency (as a reason for removing privity, so dont have
2 or more suits every time)
e. HUGE burden to find evidence for P (exclusive control of D)
iii. Introduces concept of Absolute Liability
a. Requires a P to prove only that she as actually injured by a
product, regardless of whether the product contained a defect

d. Greenman v. Yuba Power Products (p. 905, 1963) P bought power tool which
could be used in various ways, in using in one of its proper functions, small
screws fail to hold piece of wood properly and shoot it into Ps face. P sues
under negligence and breach of warranty
i. HELD: D is liable under strict products liability
a. Ds are putting products on the market knowing they will be used
without FURTHER inspection for defect.
b. NOT design defect; decided before distinctions for products
liability were in effect
e. Defect-Based
i. Traynor indicates liability should not extend to absolute
ii. Rather should turn on existence of a defect in the product
iii. SPL is not absolute
f. Restatement (Second) Section 402A
i. (1) One who sells any product in a defective condition unreasonably
dangerous to the user or consumer or to his property is subject to
liability for physical harm thereby caused to the ultimate user or
consumer, or to his property, if
a. (a) the seller is engaged in the business of selling such a product,
and
b. (b) it is expected to and does reach the user or consumer without
substantial change in the condition in which it is sold.
ii. (2) The rule stated in Subsection (1) applies although
a. (a) the seller has exercised all possible care in the preparation and
sale of his product, and
b. (b) the user or consumer has not bought the product from or
entered into any contractual relation with the seller.
g. R3 1. Liability of Commercial Seller or Distributor for Harm Caused by
Defective Products
i. One engaged in the business of selling or otherwise distributing
products who sells or distributes a defective product is subject to
liability for harm to persons or property caused by the defect.
h. What Counts as an Injury?
i. Generally for physical harm
ii. Courts hesitant in recognizing claims for things such as economic loss
iii. A particularly important question as to the injury element concerns
whether the owner of a product can invoke products liability law to

recover for a defect that causes damage to, or the destruction of, the
product itself.
iv. The general rule, subject to exceptions that vary by jurisdiction, is
that the owner cannot, and is instead left to the protections he was able
to obtain in the contract of sale via express or implied warranties. This
is sometimes dubbed the economic loss rule.
v. By Contrast: damage caused by a product defect to property other
than the product itself is ordinarily actionable in products liability
a. i.e. water heater burns down a house
i. What is a "Product"?
i. Services even when provided commercially are not products
ii. If the law were otherwise products liability could end up swallowing a
great deal of tort law
iii. Products:
a. foods and beverages, automobiles, and power tools but also to
large industrial machines, kitchen appliances, toys, recreational
and sports equipment, pharmaceuticals, chemicals sold for
commercial or residential use, certain medical devices, cosmetics
and toiletries, airplanes, boats, motorcycles, protective gear,
scientific equipment, packaging materials, farming equipment,
pesticides, chemical treatments, and so on.
iv. Note that the issue of whether something counts as a product may
come out differently if the question is being asked for some other
purpose or in some other area of the law.
v. Not Products:
a. Real property
i. i.e. land
b. Human Body parts
c. Live animals
i. Courts differ on this depending on the context in which the
injury caused by the animal comes about
ii. Dog biting a P not a product
iii. Dog sold with a disease is a product
vi. Textual material
a. i.e. encyclopedias, guides, or books
vii.
Intangibles
a. Reside at the margins
b. Generally not but occasionally yes
viii.
Used products

a. some jurisdictions deem a type of item to be a product but


nevertheless have made a deliberate decision to exempt the product
from the reach of products liability laws. Prescription drugs and
vaccines are common examples of this category.
j. Who or What is a Seller?
i. In the business of (manufacturer, distributor, commercial retailer)
ii. Not a casual seller
iii. products liability law extends far beyond manufacturers, and always
has. Indeed, as we saw from Vandermark (extending Greenman
applying defect-based liability to the retailer of a product not just
manufacturer, retailers are liable under strict products liability.
iv. i.e. consumer injured by a defective lawnmower purchased at a local
hardware store can sue store and the manufacturer
v. it is important to see that products liability law creates causes of
action against various actors within a distributive chain.
k. Gower v. Savage Arms, Inc (P 921, 2001): Man shoots himself in the foot and
brings several SPL claims
i. Rule: A manufacturer is subject to strict products liability where the
plaintiff shows that the product was defective at the time it left the
manufacturers hands and that such defect caused injury to the
plaintiff
ii. Three Kinds of Product Defect
i. P can bring multiple defects as well as negligence
b. Manufacturing Defect (this particular gun)(probably easiest for P
to establish)
i. Typical varies from every other product
ii. P establish unit that caused harm differs from prototype unit
i. Look at other product or specs
iii. Imperfection (metal ridge) prevented guns safety lock from
engaging
c. Design Defect (all Savage Model 99 guns)
i. No detent system to indicate safety lock is engaged
ii. Safety lock must be disengaged to unload gun
iii. SJ granted for : no actual causation AMOL
d. Failure to Warn (no instructions, labels)(Information flaw)
i. Lack of warning
ii. Many of these cases deal with sufficiency of warning
iii. Some about insturctions
iv. Users not warned of risk of accidental firing
v. SJ granted for : no defect & no causation AMOL

l. Proving Manufacturing Defect


i. Manufacturing Defect not always as easy as Gower
a. I.e. Escola unable to prove what was wrong with the bottle,
explains why tried to invoke res ipsa
ii. While the concept of manufacturing defect eliminates the need to
identify unreasonable conduct on the part of the defendant, it
substitutes its own question: What exactly was defective about the
product?
iii. Particularly in cases in which the harmful product has been severely
damaged or destroyed or is missing, proof problems can be daunting.
iv. In such cases, products liability plaintiffs, in a manner akin to
negligence plaintiffs who benefit from res ipsa loquitur, will usually
be given a fair bit of leeway to rely on circumstantial evidence to
prove the existence of a manufacturing defect
m. Third Restatement 3. Circumstantial Evidence Supporting Inference of
Product Defect
i. It may be inferred that the harm sustained by the plaintiff was caused
by a product defect existing at the time of sale or distribution, without
proof of a specific defect, when the incident that harmed the plaintiff:
i. (a) was of a kind that ordinarily occurs as a result of a
product defect; and
ii. (b) was not, in the particular case, solely the result of causes
other than product defect existing at the time of sale or
distribution.
b. -Authors note this most commonly applies to claims of
manufacturing defect rather than design defect or failure to warn
c. Analogy to Res Ipsa Loquitor
d. Allows for liability no RAD even though mostly rejects CET
7. DESIGN DEFECT
a. Early years of SPL: no test formally specified
i. e.g., Cronin (Cal. 1972) (jury instructed that it must find a defect to
impose liability but defect is undefined)
a. Cronin rejects s proposed instruction by which jury must find
products design rendered it unreasonably dangerous
ii. 1975-95 Move to Barker: consumer can choose R-U or CET
iii. 1995-present: R-U Dominant
a. foresight based version preferred

b. linked to requirement that P prove RAD

b. Today: dominant test is the Risk-Utility Test


i. however, there is a prominent alternative (the Consumer
Expectations Test) & there are difft versions of the R-U Test
ii. Chow v. Reckitt (P 929, 2011): moves for SJ, arguing 100% lye
drain cleaner is not defectively designed AMOL
a. Procedure: for SJ on a no-defect argument, must show U > R
i. even though at trial must prove R > U ? (Smith, J.,
concur.)
ii. NY differs from FRCP
b. Substance: a product is defectively designed if the risks associated
w/ the design exceed the designs utility
c. NYs R-U Factors
i. Utility of design to public
ii. Utility of design to
iii. Likelihood of harm associated w/ design
iv. Availability of safer design
v. Feasibility, functionality of safer design (incl. price)
vi. Consumers awareness of the designs risks
vii. MFRs ability to spread costs of safer design
d. R3d Torts, Products Liability 2(b) (1998): product is
defectively designed when the foreseeable risks of harm [it poses]
could have been reduced by the adoption of a reasonable
alternative design .
e. Proof of reasonable alternative design: relevant or reqd?
i. NY: evidence of RAD relevant but not necessary (see
factors 4 & 5)
ii. R3d: proof of RAD necessary to establish DD
f. NY jury instruction
i. Sufficient that a reasonable person who did in fact know of
the products potential for causing injury would have
concluded that the product should not have been marketed in
that condition

i. Embeds factors except #7, doesnt require them to


think about cost spreading
g. whether the R-U Test amounts to an inquiry into reasonable care,
thus returning DD cases to a negligence standard, hinges partly on
whether R-U is applied as a foresight-based test (e.g., R3d
2(b)) or a hindsight-based test (e.g., NY)
c. Consumer Expectation Test
i. was the product as safe as an ordinary consumer would expect?
ii. SPLs warranty heritage
iii. often relatively -friendly
a. Jury has broad leeway on what dangers are unexpected
iv. but sometimes -friendly
a. Difficult to claim that obvious risks are unexpected (e.g.
machine w/ unguarded feeder)
d. Wilson Sporting Goods v Hickox (P 942, 2013)
i. Test for DD is C-E not R-U (Why?)
a. C-E: did mask perform as safely as ordinary consumer would
expect?
b. Jury could reasonably answer no given evidence that/of:
i. mask concentrated rather than dissipated force
ii. other, safer masks available
iii. statements of s sales reps (best and safest)
iv. inadequate testing
c. Whether defect was reasonably foreseeable is not relevant under
C-E test: question is whether the product performed as safely as
expected
e. Soule v. General Motors Corp. (CA 1994) - P gets into car accident and wheel
crushes ankles, claims wheel design and manufacture is defective; jury is given
consumer expectations test for consideration, finds for P; D appeals because it
claims consumer expectations test should NOT have been given to jury
i. HELD: consumer expectations test use limited to when common
experience of users enables judgment of defect; HOWEVER, when
you need sophisticated (expert) knowledge, juries should be
instructed to use risk-utility test!!
a. However, court here says that since jury took expert testimony into
account, clearly risk-utility test was used, so no harm, no foul.

f. Design Defect and Prescription Drugs


i. R2d 402A, comment k
a. if properly manufactured & accompanied by adequate
warnings, a drug cannot be deemed defectively designed
i. treats drugs as unavoidably unsafe (having harmful sideeffects that cant be prevented no matter how its designed)
-Why favor this over R3?
i. Dont want to dissuade manufacturer to develop
these types of drugs
ii. Want to encourage R and D (part of the reason see
tort and state legislatures step in and say SPL not
appropriate to RXs)
ii. R3d, Products Liability 6(c)
a. a prescription drug or medical device is defectively designed only
if a reasonable MD would not prescribe it to any class of
patients
i. Note: both R2d and R3d allow for negligent design claims
i. Rejects judicially created doctrine in DD because
feels that is value of R&D for lifestyle drugs really
isnt worth it
iii. Freeman v. Hoffman-La Roche (Neb. 2000) (Accutane)
a. need only establish that drug was defective under usual Neb. test
for DD (= CE Test)
b. can prevail by showing: (1) drug was properly mfrd and had
proper warnings, (2) its benefits outweigh its risks, and (3) it could
not have been designed more safely
i. an R-U affirmative defense??
8. FAILURE TO WARN DEFECT
a. R3d, Prod. Liability 2(c)
i. A product . . . is defective because of inadequate instructions or
warnings when the 4Cble risks of harm posed by the product could
have been reduced by the provision of reasonable instructions or
warnings by the seller . . . the omission [of which] renders the product
not reasonably safe.
a. Moving Failure to Warn towards negligence

b. Warning defects (information flaw)


i. Who? Commercial sellers
ii. To Whom is duty owed: Purchaser/user
iii. When?:for non-obvious risks
iv. What should it do?: Adequately identify type and severity of risk
v. Inadequate instructions on warning when foreseeable risks of harm
posed by the product could have been reduced or avoided by the
provision or reasonable instructions or warnings
vi. Consider (1) gravity of risk, (2) likelihood of occurrence, (3) whether
the danger is commonly known, (4) if the warning will distract from
other warnings
vii.
First, establish a duty to warn (no duty for tequila, riding in
truck bed)
viii.
Warning required where
a. The danger is not obvious when the product is used as intended
i. the court in McMahon put it, the law "does not require
vendors to give warnings in the detail plaintiffs contemplate.
It expects consumers to educate themselves about the
hazards of daily life-of matches, knives, and kitchen ranges,
of bones in fish, and of hot beveragesby general reading
and experience, knowledge they can acquire before they
enter a mini mart to buy coffee for a journey." Hot coffee, in
other words, is a readily apparent risk associated with a
product much like the risks presented by drinking large
quantities of tequila (Brown Forman, p. 599), or by an open
pickup bed for hauling cargo without any passenger
restraints (Maneely, id.).
b. The intended use is not apparent or obvious (instructions)
c. Irreducible dangers (warning more likely) unless irreducible
danger is inherent to product
d. Some jx say D must have known about the defect, some jx say as
long as D did reasonable testing, its OK
ix. Heeding presumption
x. When there is a RAD, warning usually is not enough
xi. Distinguish between warning and instructions
a. Must warn of the type of danger, not just tell user what not to
do
c. Anderson v Owens-Croning (P 984, 1991): P alleges lung damage from
exposure to asbestos

i. A manufacturer must have knowledge of a potential risk or danger


inherent in its product before strict liability can be imposed for failure
to warn.
ii. must show that the risk in question was known/knowable at time of
sale
iii. reqt of known/knowable risk does not = a fault standard
iv. proof of known/knowable is necessary but not sufficient (e.g., no
duty to warn even of known risk if it is obvious)
a. i.e. Maneely: P thrown from back of pick-up truck
v. Different from first two defects, rooted in negligence
a. First two can be evaluated without reference to the conduct of the
manufacturer, giving of the warning cannot
i. Different from negligence because SL require a P to prove
only that the D did not adequately warn of a particular risk
that was known or knowable in light of the generally
recognized and prevailing best scientific and medical
knowledge available at the time of manufacture. Thus, with
SL reasonableness not a factor
d. R3:
e.
f.

g.
h.

i. If a RAD exists, warning does not absolve mfr of liability


Whether a product is not reasonably safe because lack of warning is normally
for the jury to decide
Food allergies
i. Obvious peanuts and strawberries, people will be aware and no
warning required
ii. R2 Non obvious: where substantial population is allergic and danger
is not generally know the seller is required to give warning against it
if he has knowledge or should have knowledge
iii. R3: the ingredient that causes the allergic reaction must be one whose
danger or whose presence in the product is not generally known to
consumers.When the presence of the allergenic ingredient would
not be anticipated by a reasonable user or consumer, warning
concerning its presence are required
Criticism
i. Because manufacturers just warn of everything, warnings are less
effective
Learned intermediary
i. is a defense doctrine used in the legal system of the United States.
This doctrine states that a manufacturer of a product has fulfilled his
duty of care when he provides all of the necessary information to a

"learned intermediary" who then interacts with the consumer of a


product.
i. Heeding presumption:
i. Failure to warn is different than first two defects because requires
cause showing raises the question of whether someone would have
selected a different course of conduct if information had been
provided
ii. Motus
a. Introduces Heeding presumption that if they knew would P have
altered action
i. Adopted by R2 but rejected by R3 which says to apply
prevailing rules and principle of tort causation
j. Defenses
i. State of limitations
ii. Regulatory compliance (not really a defense)
iii. Contributory/comparative negligence (if based on negligence)
iv. Assumption of risk (if based on SL) treated like comparative fault
v. Product misuse (manufacturer SL for foreseeable misuse) not
really a defense, prevents P from establishing a PF case (cant
establish that the product was defective)
vi. Open and obvious dangers for products with clear dangers that
cannot be designed out (e.g. knives and cuts)
vii.
Lack of proximate cause
viii.
State of the art given scientific knowledge at the time product
was sold
k. Damages
i. Punitive damages are usually not allowed in DD cases because it isnt
usually willful or malicious
ii. No limit on punitives for the same activities either, S.Ct has only said
evidence of injuries to other people are not allowed
a. Exception Grimshaw v. Ford: Pinto was designed purposefully to
keep price and weight at $2000 and gas tank blew up and injured P
Basis for
Liability

Scope

Decision
Mode

Duty

Decisionmaker

Burden of
proof

Negligence

based on fault

B>p*L?

case-by-case

duty is a
necessary
element

jury must
decide breach

Plaintiff

Classic Strict
Liability

imposes
liability even
where no party
is at fault
high risk and
conventional
use?

categorical

duty is not
required for
physical harm

judge
Decides

plaintiff

Products
Liability

Where product
defect, based
on risk/utility
balancing

Product
specific can
be categorical

No duty
required under
McPherson
can extend to
emotional
damages for
consumers

Jury

defendant but
only once
plaintiff has
established a
deviation
(MD), a RAD
(DD), no
warning for
product that is
not obviously
dangerous
(failure to
warn)

You might also like